Вы находитесь на странице: 1из 85

Copyright 2014 Delhi Academy of Medical Sciences, All Rights Reserved.

1/85
Test Information
Test Name SWTS - MEDICINE - 1 Total Questions 200

Test Type Examination Difficulty Level Difficult

Total Marks 600 Duration 120minutes

Test Question Language:- ENGLISH

(1). A 21 year old woman is known to suffer from anorexia nervosa. Which of the following metabolic disturbances would be a characteristic
finding?

a. A decrease in Cortisol levels

b. Hyperkalaemia

c. Impaired glucose tolerance

d. Raised androgen levels

Solution. Ans 1: (c) Impaired glucose tolerance


Ref Read the text below
Sol:
Malnutrition is another cause of diabetes.
Cortisol and growth hormone levels are elevated.
LH and FSH would be low and LH response to LHRH is impaired when weight loss is severe.
Hypokalaemia (not hyperkalaemia may be seen).
Also there may be hypoalbuminaemia, anaemia, leukopenia, and raised serum carotene.

Correct Answer. c

(2). All of the following are seen in cardiac tamponade except :

a. Pulsus paradoxus

b. Diastolic collapse of right ventricle on echocardiogram

c. Electrical alternans

d. Kussmauls sign

Solution. Ans-2: (d) Kussmauls sign


Ref: Read the text below
Sol :
CLINICAL FEATURES OF CARDIAC TAMPONADE :
Pulsus Paradoxus Common
o Jugular veins.
o Prominent y descent absent.
o Prominent x descent present.
Kussmauls Sign absent
o Electrocardiogram
o Low ECG voltage may be present.
o Electrical alternans may be present.
ECG:
Low ECG voltage.
Electrical alterans`

Correct Answer. d

Copyright 2014 Delhi Academy of Medical Sciences, All Rights Reserved. 2/85
(3). A 45-year-old male with a history of hypertrophic cardiomyopathy presents with severe dyspnea and bilateral inspiratory rales. What is
the appropriate pharmacologic management?

a. Nitroglycerin

b. Nesiritide

c. Morphine

d. -adrenergic blocker.

Solution. 3: (d) -adrenergic blocker


Ref Read the text below
Sol:
Hypertrophic cardiomyopathy is characterized by asymmetric left ventricular hypertophy resulting in impaired relaxation and left
ventricular outflow tract (LVOT) obstruction.
Maneuvers or pharmacologic agents that decrease preload (or increase chronotropy) will increase LVOT obstruction, thus worsening
pulmonary congestion.
The cornerstone of therapy is decreased chronotropy to allow for increased diastolic filling time of the noncompliant left ventricle.
The end result is a decrease in both LVOT obstruction and pulmonary venous congestion.
Nitroglycerin, nesiritide, and morphine will cause a reduction in preload and thus likely worsen the patient's clinical status

Correct Answer. d

(4). Which of the following electrocardiogram findings, occurring shortly after the onset of acute myocardial infarction, is associated with an
increased mortality?

a. Second-degree Mobitz type I atrioventricular block.

b. First-degree atrioventricular block.

c. New right bundle branch block.

d. Sinus bradycardia.

Solution. Ans 4: (c) New right bundle branch block


Ref Read the text below
Sol:
New-onset right bundle branch block is most commonly seen in the setting of an anteroseptal myocardial infarction. It portends an
increase in mortality as it often leads to complete heart block.
First-degree atrioventricular block is usually benign. Sinus bradycardia is not associated with an increased mortality in the absence of
hemodynamic instability and may be protective by reducing myocardial oxygen demand.
Second-degree Mobitz type I (Wenckebach) is the most common form of second-degree atrioventricular block occurring in the setting of
acute myocardial infarction. It rarely progresses to complete heart block.

Correct Answer. c

(5). In which of the following conditions would it be expected to find an elevated plasma total cortisol concentration?

a. Congenital adrenal hyperplasia

b. Patients on long-term benzodiazepine therapy

c. Patients taking prednisolone

d. Pregnancy

Solution. Ans 5: (d) Pregnancy


Ref Read the text below
Sol:
Cortisol levels are increased in pregnancy, conditions of physical and emotional stress and drug therapy (oestrogens, oral
contraceptives, amphetamines, cortisone, and spironolactone).
Treatment with other forms of steroid lead to decreased levels of cortisol.

Correct Answer. d

Copyright 2014 Delhi Academy of Medical Sciences, All Rights Reserved. 3/85
(6). Which of the following statements are true of primary hyperparathyroidism?

a. It is associated with hypocalciuria due to elevated PTH levels.

b. PTH is secreted in a pulsatile manner from the posterior pituitary and acts through PTH receptors on parathyroid cell membranes

c. It is usually caused by an adenoma of a single parathyroid gland.

d. It progresses to tertiary hyperparathyroidism with time.

Solution. Ans 6: (c) It is usually caused by an adenoma of a single parathyroid gland.


Ref Read the text below
Sol:
"Primary HPT can be divided pathologically into adenoma, hyperplasia, and carcinoma. Adenomas clearly are the most prevalent entity
representing 80-85% of cases.
Hyperplasia is the second most common diagnosis constituting 15% of cases. Carcinoma represents <1% of total cases.
Double adenoma has been found in approximately 5% of the time, and complicates the clinical distinction between adenoma and
hyperplasia.
Histologically, normal parathyroid tissue shows a cell to fat ratio of 1:1. Hypercellular parathyroid tissue is typified by the loss of the
normal amount of fat.
In primary hyperparathryroidism there is usually hypercalciuria. Secondary hyperparathyroidism may progress to tertiary but primary
does not.

Correct Answer. c

(7). A previously fit, 30-year-old female presents with a four day history of intractable pruritus and urticaria. What is the most appropriate
initial management?

a. Chlorpheniramine

b. Ranitidine

c. Prednisolone

d. Topical hydrocortisone

Solution. Ans 7: (a) Chlorpheniramine


Ref Read the text below
Sol:
Urticaria is a common condition and usually responds very well to systemic antihistamines which the correct first line treatment.
Oral steroids can be given for severe cases but only as a last resort and topical steroids/ topical antihistamines have no effect.

Correct Answer. a

Copyright 2014 Delhi Academy of Medical Sciences, All Rights Reserved. 4/85
(8). A 36-year-old male with insulin-dependent diabetes mellitus of three years duration presented with decreased libido and erectile
dysfunction since diagnosis. No abnormalities were noted on genital examination. Investigations revealed:
Plasma testosterone 6.0 nmol/L (9 - 35)
Plasma follicle stimulating hormone 1.0 u/L (1-8)
Which of the following investigations is most appropriate next step?

a. Autonomic function testing

b. Doppler studies of penile artery

c. Nerve conduction studies

d. Serum ferritin

Solution. Ans 8: (d) Serum ferritin


Ref Read the text below
Sol:
This IDDM patient appears to have hypogonadotrophic hypogonadism (HH) as reflected by low testosterone and low FSH.
The combination is compatible with a diagnosis of haemochromatosis and measuring ferritin would be a reasonable investigation.
Haemochromatosis typically causes hypogonadotrophic hypogonadism as a consequence of the ferritin deposition within the pituitary
rather than primary testicular dysfunction. Autonomic nerve dysfunction is one of the commoner causes of impotence in a diabetic but in
this case is not the cause of his HH.
For similar reasons, both nerve conduction studies and dopplers are irrelevant.
Prolactin would be a sensible measurement but probably if you were looking to confirm a diagnosis that incorporates the diabetes as
well, Ferritin would be the investigation of choice.

Correct Answer. d

(9). All are true about temporal arteritis except

a. Can lead to sudden bilateral blindness

b. More common in females

c. Worsens on exposure to heat

d. Mostly affects elderly

Solution. Ans-9: (c) Worsens on exposure to heat


Ref: Read the text below
Sol:
Temporal (giant cell) arteritis is an inflammatory disorder of arteries that frequently involves the extracranial carotid circulation.
It is a common disorder of the elderly
The average age of onset is 70 years, and women account for 65% of cases.
About half of patients with untreated temporal arteritis develop blindness due to involvement of the ophthalmic artery and its branches;
indeed, the ischemic optic neuropathy induced by giant cell arteritis is the major cause of rapidly developing bilateral blindness in
patients >60 years.
Because treatment with glucocorticoids is effective in preventing this complication, prompt recognition of the disorder is important.
Typical presenting symptoms include headache, polymyalgia rheumatica ,jaw claudication, fever, and weight loss.
Headache is the dominant symptom and often appears in association with malaise and muscle aches.
Head pain may be unilateral or bilateral and is located temporally in 50% of patients but may involve any and all aspects of the cranium.
Pain usually appears gradually over a few hours before peak intensity is reached; occasionally, it is explosive in onset.
The quality of pain is only seldom throbbing; it is almost invariably described as dull and boring, with superimposed episodic stabbing
pains similar to the sharp pains that appear in migraine.
Most patients can recognize that the origin of their head pain is superficial, external to the skull, rather than originating deep within the
cranium (the pain site for migraineurs). Scalp tenderness is present, often to a marked degree; brushing the hair or resting the head on a
pillow may be impossible because of pain.
Headache is usually worse at night and often aggravated by exposure to cold. Additional findings may include reddened, tender nodules
or red streaking of the skin overlying the temporal arteries, and tenderness of the temporal or, less commonly, the occipital arteries.
The erythrocyte sedimentation rate (ESR) is often, though not always, elevated; a normal ESR does not exclude giant cell arteritis.
A temporal artery biopsy followed by treatment with prednisone 80 mg daily for the first 46 weeks should be initiated when clinical
suspicion is high.

Correct Answer. c

Copyright 2014 Delhi Academy of Medical Sciences, All Rights Reserved. 5/85
(10). Amyloidosis deposition most commonly occurs in :

a. Renal vessels

b. Knee joints

c. Skin

d. Cornea

Solution. Ans-10: (a) Renal vessels


Ref.: Ch. 6 Disease of immune system - 254
Sol :
Amyloid is abnormal pathological protein resulting from improper folding of proteins and deposited extracellularly as fibrils and disrupt
their normal function.
The most common site of deposition is kidney, followed by, liver, spleen, lymph nodes, adrenals, thyroid, GIT, heart etc.
In systemic amyloidosis, any organ can show deposition of Amyloid. Among all organs, kidneys are most commonly and most seriously
get affected by Amyloidosis.

Correct Answer. a

(11). Mortality ratio in cerebral malaria is

a. 10-20%

b. 20-30%

c. 30-50%

d. 50-70%

Solution. Ans-11: (c) 30-50%


Ref:Read the text below
Sol:
Cerebral malaria (CM) collectively involves the clinical manifestations of Plasmodium falciparum malaria that induce changes in mental
status and coma.
It is an acute, widespread disease of the brain which is accompanied by fever. The mortality ratio is between 25-50%.( so choice C is
closest) If a person is not treated, CM is fatal in 24-72 hours.
The histopathological hallmark of this encephalopathy is the sequestration of cerebral capillaries and venules with parasitized red blood
cells (PRBCs) and non-PRBCs (NPRBCs). Ring-like lesions in the brain are major characteristics.
Disease risk factors include being a child under 10 years of age and living in malaria-endemic area.

Correct Answer. c

(12). What is the leading cause of death in patients with infective endocarditis?

a. Congestive heart failure.

b. Sepsis

c. Meningitis

d. Pneumonia

Solution. Ans 12: (a) Congestive heart failure


Ref Read the text below
Sol:
Acute or progressive congestive heart failure (CHF) occurs in up to 70% of patients with infective endocarditis (IE).
The extracardiac manifestations of IE are usually the result of arterial embolization of fragments of the friable vegetation and are second
to CHF as the leading cause of complications of IE.

Correct Answer. a

Copyright 2014 Delhi Academy of Medical Sciences, All Rights Reserved. 6/85
(13). Which of the following is a characteristic of primary hypothyroidism?

a. Absence of pubic hair

b. Soft skin.

c. Amenorrhea

d. Cardiomegaly

Solution. Ans 13: (d) Cardiomegaly


Ref Read the text below
Sol:
The most common etiologies of hypothyroidism are primary thyroid failure due to autoimmune diseases, idiopathic causes, ablative
therapy, and iodine deficiency.
Features of primary hypothyroidism include previous thyroid operation, obesity, hypothermia, coarse voice, pubic hair unchanged, dry
coarse skin, increased heart size, normal menses and lactation, normal sella turcica, increased thyroid-stimulating hormone (TSH),
normal plasma cortisol, no response to TSH, and good response to levothyroxine without steroids.

Correct Answer. d

(14). Which of the following is not true of aortic dissection?

a. Commonly presents with pulse discrepancies in the upper extremeties.

b. May produce tamponade.

c. May produce cerebrovascular symptoms.

d. Commonly presents with a tearing pain.

Solution. Ans 14: (a) Commonly presents with pulse discrepancies in the upper extremeties.
Ref Read the text below
Sol:
While pulse discrepancies can occur in aortic dissection, they are present in <20% of patients. Pulse discrepancies do not exist with
abdominal aortic aneurysms.
Tamponade and cerebrovascular symptoms are wellknown complications of dissection.
The pain of aortic dissection is a tearing or ripping type of pain that is abrupt in onset.

Correct Answer. a

(15). Which of the following commonly prescribed medications decreases serum phenytoin levels?

a. Warfarin

b. Azithromycin

c. Folic acid

d. Cimetidine

Solution. Ans 15: (c) Folic acid.


Ref Read the text below
Sol:
Any medication that either inhibits or enhances hepatic microsomal activity can increase or decrease (respectively) serum phenytoin
levels.
Drug interactions are a common scenario when a known seizure patient, who has been compliant on phenytoin, presents with a seizure
and a decreased phenytoin level. Folic acid enhances phenytoin metabolism in the liver and thus can cause decreased serum phenytoin
levels.
Wafarin and cimetidine inhibit the cytochrome P450 system in the liver and thus cause increased levels of phenytoin.
Azithromycin does not affect phenytoin metabolism significantly.
Some other medications displace phenytoin from protein binding sites, resulting in a decreased serum level but an increased free
phenytoin fraction. This group includes sulfonamides, valproic acid,and salicylates

Correct Answer. c

Copyright 2014 Delhi Academy of Medical Sciences, All Rights Reserved. 7/85
(16). A patient taking clozapine should have which of the following checked at regular intervals?

a. Thyroid function tests

b. White blood cell count.

c. Serum cortisol

d. Platelet count

Solution. Ans 16: (b) White blood cell count


Ref Read the text below
Sol:
Patients taking clozapine risk agranulocytosis, and those presenting to the ED with fever or other infectious complaints should have a
white blood cell count.

Correct Answer. b

(17). A 35-year-old male presents with a gunshot wound to his right chest. He has hemoptysis on presentation. Just after endotracheal
intubation, he goes into cardiac arrest. What is the most likely diagnosis?

a. Air embolism.

b. Pericardial tamponade.

c. Esophageal intubation

d. Massive hemothorax

Solution. Ans 17: (a) Air embolism


Ref Read the text below
Sol:
In patients with lung injury, systemic air embolism is a cause of cardiac arrest after endotracheal intubation.
The diagnosis should be suspected in patients with penetrating chest wounds who have hemoptysis.
Positive pressure ventilation can result in air being forced from an injured bronchus into an adjacent injured vessel, producing a venous
air embolism.
These patients develop hemodynamic compromise and dysrhythmias with intubation and ventilation.
Pericardial tamponade is possible but less likely with a right-sided injury. Massive hemothorax may cause hemodynamic instability but
would not necessarily be related to intubation.
Esophageal intubation would result in hypoxia and bradycardia before cardiac arrest

Correct Answer. a

(18). Patient presented within chest pain to emergency room and dies within 12 hours of arrival. After 5 days, autopsy shows necrosis in
Myocardium. What is the type of necrosis?

a. Coagulative

b. Fibrinoid

c. Caseous

d. Liquefactive

Solution. Ans-18: (a) Coagulative necrosis


Ref.: Robbins and cotran pathologic Basis of Disease : 8th ed.
Sol :
Organs with end arterial blood supply will show usually coagulative type of necrosis; solid organs like heart will always show coagulative
necrosis.
Caseous necrosis is usually seen in tuberculosis and shows associated granulomatous inflammation.
Liquefactive necrosis is usually seen in brain parenchyma.
Fibrinoid necrosis is special form of necrosis seen in immune complex mediated destruction of blood vessels like in various types of
vasculitis.

Correct Answer. a

Copyright 2014 Delhi Academy of Medical Sciences, All Rights Reserved. 8/85
(19). Which of the following is not a part of metabolic syndrome?

a. Central obesity

b. Hypertriglyceridemia

c. Hypertension

d. Myocardial infarction

Solution. Ans-19: (d) Myocardial infarction


Ref.: Read the text below
Sol :
The metabolic syndrome (syndrome X, insulin resistance syndrome) consists of a constellation of metabolic abnormalities that confer
increased risk of cardiovascular disease (CVD) and diabetes mellitus (DM).
The criteria for the metabolic syndrome have evolved since the original definition by the World Health Organization in 1998, reflecting
growing clinical evidence and analysis by a variety of consensus conferences and professional organizations.
The major features of the metabolic syndrome include central obesity, hypertriglyceridemia, low HDL cholesterol, hyperglycemia, and
hypertension.

Correct Answer. d

(20). Infection associated with Atherosclerosis is?

a. Chlymadia trachomatis

b. Chlymadia pneumoniae

c. Chlymadia psittaci

d. Dhlymadia gingivalis

Solution. Ans-20: (b) Chlymadia pneumoniae


Ref.: Read the text below
Sol :
Epidemiologic studies have demonstrated an association between serologic evidence of C. pneumoniae infection and atherosclerotic
disease of the coronary and other arteries.

Correct Answer. b

(21). Physiological murmur is associated with all except :

a. Only diastolic murmur

b. Midsystolic murmur

c. Position dependent

d. Not audible without stethoscope

Solution. Ans-21: (a) Only diastolic murmur


Ref.: Read the text below
Sol :
A functional murmur (innocent murmur, physiologic murmur) is a heart murmur that is primarily due to physiologic conditions outside
the heart, as opposed to structural defects in the heart itself.

Correct Answer. a

Copyright 2014 Delhi Academy of Medical Sciences, All Rights Reserved. 9/85
(22). Which of the following medications should be considered as a first-line agent in a 68-year-old male presenting with a blood pressure of
70/45 mmHgafter an ST elevation myocardial infarction?

a. Dopamine

b. Dobutamine

c. Norepinephrine

d. Milrinone

Solution. Ans 22: (a) Dopamine.


Ref Read the text below
Sol:
Dopamine is a vasopressor for which dose-dependent response is expected. Patients in acute cardiogenic shock have decreased
perfusion and, depending on the extent of myocardial dysfunction, may not respond adequately to fluid boluses.
Hemodynamic monitoring in these patients is recommended.Dobutamine is a positive inotrope that can decrease blood pressure.
The use of dobutamine should be carefully titrated. It is not prudent to use dobutamine in patients with hypotension (systolic <90
mmHg).
Norepinephrine is a potent vasopressor with primarily 1 agonist properties and can be used if the patient's hemodynamics are not
responding to dopamine.
Milrinone, a phosphodiesterase inhibitor, has been used for its inotropic properties, but because it can also lower blood pressure
through its effects on systemic vascular resistance it should be used in concert with a vasopressor.

Correct Answer. a

(23). A 25-year-old male sustained an amputation of his left forearm while using a table saw. He has a heart rate of 130 beats per minute and a
systolic blood pressure of 90/40 mmHg. What class of hemorrhage is this patient in?
.

a. Class I

b. Class II.

c. Class III

d. Class IV

Solution. Ans 23: (c) Class III.


Ref Read the text below
Sol:
As detailed in Table this patient has a class III hemorrhage.
Although this is a somewhat arbitrary classification of hemorrhage, it helps differentiate the amount of blood

loss based on vital signs.


Class III shock represents 30-40% blood loss and is manifested as hypotension and tachycardia.
ESTIMATED FLUID AND BLOOD LOSSES BASED ON PATIENT'S INITIAL
PRESENTATION

Correct Answer. c

Copyright 2014 Delhi Academy of Medical Sciences, All Rights Reserved. 10/85
(24). Which of the following is not an appropriatetreatment for acute mountain sickness?

a. Acetazolamide 250 mg PO q8h.

b. Dexamethasone 4 mg PO q6h.

c. Oxygen 2-4 L/min.

d. Nifedipine 20 mg q12h of extended release formulation

Solution. Ans 24: (d) Nifedipine 20 mg q12h of extended release formulation.


Ref Read the text below
Sol:
Acute mountain sickness (AMS) may be treated by descent to a lower altitude or acclimatization to the same
altitude if the illness is mild.
Oxygen promptly relieves most symptoms of AMS. Acetazolamide and dexamethasone are pharmacologic
agents that may be used as an alternative to descent in mild to moderate cases and as adjunctive treatment to
descent in severe AMS or high altitude cerebral edema (HACE).
Nifedipine may be useful in treatment of high altitude pulmonary edema but is not effective for AMS or HACE.

Correct Answer. d

(25). What is the most common cause of a unilateral neck mass in patients older than 40 years?

a. Reactive lymphadenopathy.

b. Hodgkin's disease.

c. Squamous cell carcinoma.

d. Salivary gland infection.

Solution. Ans 25: (c) Squamous cell carcinoma


Ref Read the text below
Sol:
In patients older than 40 years, 75% of neck masses are neoplastic. The most common cause of a unilateral neck
mass is squamous cell carcinoma.
Masses can be initially mobile and then fixed as the cancer invades surrounding tissue.
Other tumors presenting unilaterally are neoplasms of the salivary gland and thyroid and lymphomas.
They can become superinfected and present as abscesses.

Correct Answer. c

Copyright 2014 Delhi Academy of Medical Sciences, All Rights Reserved. 11/85
(26). The standard starting regimen for acid-fast bacilli smearpositive active pulmonary tuberculosis is

a. Isoniazid

b. Isoniazid, rifampin

c. Isoniazid, moxifloxacin, pyrazinamide, ethambutol

d. Isoniazid, rifampin, pyrazinamide, ethambutol

Solution. Ans 26: (d) Isoniazid, rifampin, pyrazinamide, ethambutol Ref Read the text below
Sol:
Drugs used to treat Mycobacterium tuberculosis are classified as first-line or second-line. First-line agents,
which are proven most effective and are necessary for any short-course treatment regimen, include isoniazid,
rifampin, ethambutol,and pyrazinamide.
First-line supplemental agents, which are highly effective with acceptable toxicity, include rifabutin,
rifapentine, and streptomycin.
Second-line agents,which are either less clinically active or have greater toxicity, include para-aminosalicylic acid,
ethionamide, cycloserine, amikacin, and capreomycin. The fluoroquinolones, levofloxacin and moxifloxacin, are
active against M. tuberculosis but are not yet considered first-line therapy.
Some experts consider moxifloxacin a supplemental first-line therapy. It is necessary to have at least three active
agents during the 2-month induction phase of active tuberculosis therapy. Ethambutol is initially used as a fourth
agent to account for the possibility of drug resistance to one of the other agents.
Consolidation phase includes rifampin and isoniazid, and is 47 months in length, depending on anatomic
location of infection as well as clearance of sputum cultures at 2 months.

Correct Answer. d

(27). Which of the following is true regarding the diagnosis of Legionella pneumonia?

a. Acute and convalescent antibodies are not helpful due to the presence of multiple serotypes.

b. Legionella can never be seen on a Gram stain.

c. Legionella cultures grow rapidly on the proper media.

d. Legionella urine antigen is detectable within 3 days of symptoms and is not affected by antibiotic use

Solution. Ans 27: (d) Legionella urine antigen is detectable within 3 days of symptoms and is not affected by antibiotic use
Ref Read the text below
Sol:
Legionella urine antigen is detectable within 3 days of symptoms and will remain positive for 2 months. It is not affected by antibiotic
use.
The urinary antigen test is formulated to detect only L. pneumophilia (which causes 80% of Legionella infections) but cross-reactivity
with other Legionella species has been reported.
The urinary test is sensitive and highly specific. Typically, Grams staining of specimens from sterile sites such as pleural fluid show
numerous white blood cells but no organisms.
However, Legionella may appear as faint, pleomorphic gram-negative bacilli. Legionella may be cultured from sputum even when
epithelial cells are present. Cultures, grown on selective media, take 35 days to show visible growth. Antibody detection using acute and
convalescent serum is an accurate means of diagnosis.
A fourfold rise is diagnostic, but this takes up to 12 weeks so is most useful for epidemiologic investigation.
Legionella PCR has not been shown to be adequately sensitive and specific for clinical use. It is used for environmental sampling.

Correct Answer. d

Copyright 2014 Delhi Academy of Medical Sciences, All Rights Reserved. 12/85
(28). Which of the following medications used as antimycobacterial drugs require dose reduction for patients with an estimated glomerular
filtration rate <30 mL/min?

a. Isoniazid

b. Pyrazinamide

c. Rifabutin

d. Streptomycin

Solution. Ans 28: (d) Streptomycin


Ref Read the text below
Sol:
Streptomycin is an aminoglycoside first-line supplemental agent for treatment of Mycobacterium tuberculosis, M. marinum, and M.
kansasii.
It is only available for IM or IV use and is not commonly utilized in the United States because of toxicity. Adverse reactions occur in
1020% of patients. Renal toxicity and ototoxicity are most common.
Ototoxicity may involve hearing and vestibular function. Like other aminoglycosides, it is eliminated almost exclusively by renal
mechanisms, so drug levels must be followed along with renal function.
Isoniazid and rifampin must be avoided in patients with severe hepatic toxicity. Pyrazinamide is also metabolized by liver and should be
used carefully in patients with liver disease.

Correct Answer. d

(29). Which of the following statements regarding the currently licensed human papillomavirus (HPV) vaccine (Gardasil) is true?

a. It does not protect against genital warts.

b. It is an inactivated live virus vaccine.

c. Once sexually active, women will derive little protective benefit from the vaccine.

d. Vaccinees should continue to receive standard Pap smear testing

Solution. Ans 29: (d) Vaccinees should continue to receive standard Pap smear testing.
Ref Read the text below
Sol:
There will soon be two available HPV vaccines. Gardasil (Merck) is currently licensed and contains HPV types 6, 11, 16, and 18; Cervarix
(Glaxo- SmithKline) is pending final regulatory approval and contains HPV types 16 and 18. HPV types 6 and 11 cause 90% of anogenital
warts.
HPV 16 and 18 cause 70% of cervical cancers. Both vaccines consist of virus-like particles without any viral nucleic acid, therefore are
not active.
Both provide nearly 100% protection against two common oncogenic strains of HPV (16 and 18) but neglect to cover the other strains
that cause up to 30% of cervical cancer.
Because the vaccines do not protect against all oncogenic HPV serotypes, it is recommended that Pap screening of women for cervical
cancer continue according to prior schedules.
The vaccine should be given to girls and young women between the ages of 9 and 26 provided that they do not have evidence of
infection with both HPV 16 and 18 already.

Correct Answer. d

Copyright 2014 Delhi Academy of Medical Sciences, All Rights Reserved. 13/85
(30). In a patient with bacterial endocarditis, which of the following echocardiographic lesions is most likely to lead to embolization?

a. 5-mm mitral valve vegetation

b. 5-mm tricuspid valve vegetation

c. 11-mm aortic valve vegetation

d. 11-mm mitral valve vegetation

Solution. Ans 30: (d) 11-mm mitral valve vegetation


Ref Read the text below
Sol:
While any valvular vegetation can embolize, vegetations located on the mitral valve and vegetations >10 mm are greatest risk of
embolizing.However, only choice D demonstrates the risks of both size and location.
Hematogenously seeded infection from an embolized vegetation may involve any organ, but particularly affects those organs with the
highest blood flow. They are seen in up to 50% of patients with endocarditis.
Tricuspid lesions will lead to pulmonary septic emboli,common in injection drug users. Mitral and aortic lesions can lead to embolic
infections in the skin, spleen, kidneys, meninges, and skeletal system.

Correct Answer. d

(31). Abacavir is a nucleoside transcription inhibitor that carries which side effect unique for HIV antiretroviral agents?

a. Fanconis anemia

b. Granulocytopenia

c. Lactic acidosis

d. Severe hypersensitivity reaction.

Solution. Ans 31: (d) Severe hypersensitivity reaction


Ref Read the text below
Sol:
Abacavir use is associated with a potentially severe hypersensitivity reaction in about 5% of patients. There is likely a genetic
component, with HLA-B*5701 being a significant risk factor for hypersensitivity syndrome.
Symptoms,which usually occur within 2 weeks of therapy but can take >6 weeks to emerge, include fever, maculopapular rash, fatigue,
malaise, gastrointestinal symptoms, and/or dyspnea.Once a diagnosis is suspected, the drug should be stopped and never given again
because rechallenge can be fatal.
For this reason, both the diagnosis and patient education once the diagnosis is made must be performed thoroughly and carefully. It is
important to note that two available combination pills contain abacavir (epzicom, trizivir), so patients must know to avoid these as well.
Fanconis anemia is a rare disorder associated with tenofovir. Zidovudine causes anemia and sometimes granulocytopenia. Stavudine
and other nucleoside reverse transcriptase inhibitors are associated with lipoatrophy of the face and legs.

Correct Answer. d

Copyright 2014 Delhi Academy of Medical Sciences, All Rights Reserved. 14/85
(32). A patient is evaluated for peripheral cyanosis. Which of the following is not a potential etiology?

a. Cold exposure

b. Deep venous thrombosis

c. Methemoglobinemia

d. Peripheral vascular disease

Solution. Ans 32: (c) Methemoglobinemia


Ref Read the text below
Sol:
In the evaluation of cyanosis, the first step is to differentiate central from peripheral cyanosis.
In central cyanosis, because the etiology is either reduced oxygen saturation or abnormal hemoglobin, the physical findings include
bluish discoloration of both mucous membranes and skin.
In contrast, peripheral cyanosis is associated with normal oxygen saturation but slowing of blood flow and an increased fraction of
oxygen extraction from blood; subsequently, the physical findings are present only in the skin and extremities.
Mucous membranes are spared. Peripheral cyanosis is commonly caused by cold exposure with vasoconstriction in the digits. Similar
physiology is found in Raynauds phenomenon. Peripheral vascular disease and deep venous thrombosis result in slowed blood flow and
increased oxygen extraction with subsequent cyanosis.
Methemoglobinemia causes abnormal hemoglobin that circulates systemically. Consequently, the cyanosis associated with this disorder
is systemic.
Other common causes of central cyanosis include severe lung disease with hypoxemia, right-to-left intracardiac shunting, and pulmonary
arteriovenous malformation

Correct Answer. c

(33). All the following drugs can cause eosinophilic pneumonia except

a. Nitrofurantoin

b. Sulfonamides

c. Isoniazid

d. Amiodarone

Solution. Ans 33: (d) Amiodarone


Ref Read the text below
Sol:
Multiple drugs have been associated with eosinophilic pulmonary reactions. They include nitrofurantoin, sulfonamides, NSAIDs,
penicillins, thiazides, tricyclic antidepressants, hydralazine, and chlorpropramide, among others.
Amiodarone can cause an acute respiratory distress syndrome with the initiation of the drug as well as a syndrome of pulmonary
fibrosis.
Eosinophilic pneumonia is not caused by amiodarone

Correct Answer. d

Copyright 2014 Delhi Academy of Medical Sciences, All Rights Reserved. 15/85
(34). All of the following support the diagnosis of streptococcal pharyngitis except

a. Cough

b. Fever

c. Pharyngeal exudates

d. Positive rapid streptococcal throat antigen test

Solution. Ans 34: (a) Cough


Ref Read the text below
Sol:
Streptococcus pyogenes is the most common cause of bacterial pharyngitis in adults, accounting for ~515% of cases of acute
pharyngitis (the largest number being viral).
Group A Streptococcus is an uncommon cause of pharyngitis after age 15. Cough and coryza are more suggestive of viral pharyngitis, as
is a less severe sore throat
Pharyngeal exudates, tender cervical adenopathy, fever, and lack of cough are all more predictive of pharyngitis due to S. pyogenes.
Some experts recommend empirical penicillin treatment without throat sampling for rapid antigen and culture if at least three or four of
the above clinical criteria are met, while others recommend making a microbiologic diagnosis in all cases where streptococcal infection
is being considered.
The rapid streptococcal antigen test is indeed rapid but lacks complete sensitivity in a clinic setting. Sending streptococcal
antigennegative samples for culture that is more sensitive but takes 24 days to return is also controversial.

Correct Answer. a

(35). Most common cause of refractory hypertension is?

a. Noncompliance of patient

b. Obstructive sleep apnea

c. Pheochromocytoma

d. Renovascular disease

Solution. Ans-35: (b) Obstructive sleep apnea


Ref.: Read the text below
Sol :
The most common secondary causes of resistant hypertension are obstructive sleep apnea, hyperaldosteronism, renal parenchymal
disease and renal artery stenosis.
Renovascular hypertension may cause resistant hypertension but is less common than sleep apnea.
Hyperaldosteronism has been fond in as many as 20 percent of patients with resistant hypertension

Correct Answer. b

(36). Which of the following is a Sino atrial disease?

a. Atrial ectopic

b. Ventricular ectopic

c. Sinus arrest

d. All of the above

Solution. Ans-36: (c) Sinus arrest


Ref.: Read the text below
Sol :
SA node dysfunction may be completely asymptomatic and manifest as an ECG anomaly such as sinus bradycardia; sinus arrest and exit
block; alternating supraventricular tachycardia, usually atrial fibrillation, and bradycardia.

Correct Answer. c

Copyright 2014 Delhi Academy of Medical Sciences, All Rights Reserved. 16/85
(37). Loud P2 is found in ?

a. Pulmonary HTN

b. MS

c. MR

d. Aotic incompetence

Solution. Ans-37: (a) Pulmonary HTN


Ref.: Read the text below
Sol :
Loud P2
Pulmonary hypertension unless proved otherwise
Eisenmengers syndrome due to ASD, VSD, PDA
Multiple pulmonary thrombi

Correct Answer. a

(38). Fainting attacks common in which conditions?

a. Aortic regurgitation

b. Aortic stenosis

c. Mitral regurgitation

d. Mitral stenosis

Solution. Ans-38: (b) Aortic stenosis


Ref.: Read the text below
Sol :
Exertional dyspnea, angina pectoris, and syncope are the three cardinal symptoms of aortic stenosis.
AS is rarely of clinical importance until the valve orifice has narrowed to approximately 1 cm2.
Even severed AS may exist for many years without producing any symptoms because of the ability of the hypertrophied LV to generate
the elevated intraventricular pressures required to maintain a normal stroke volume.
Once symptoms occur, valve replacement is indicated

Correct Answer. b

(39). LAP score is decreased in:

a. CML

b. Polycythemia

c. Pregnancy

d. Bacterial infection

Solution. Ans-39: (a) CML


Ref.: Read the text below
Sol :
AP is leukocyte alkaline phosphatase, the enzyme score which is used sometimes to distinguish leukemoid reactions (usually from
bacterial infections) and CML. The LAP will be present in the toxic granules of neutrophils and levels are usually raised with bacterial
infections. Hence, LAP score will be less in CML, when compared to Leukemoid reactions. It has no role in polycythemia or in pregnancy.
Historically, various clues including the leukocyte alkaline phosphatase score and the presence of basophilia were used to distinguish
CML from a leukemoid reaction. However, at present the test of choice in adults to distinguish CML from a leukemoid reaction.
However, at present the test of choice in adults to distinguish CML is an assay for the presence of the Philadelphia chromosome, either
via cytogenetics and FISH, or via PCR for the BCR/ABL fusion gene. The LAP (Leukocyte Alkaline Phophatase) score is high in reactive
states but is low in CML. In cases where the diagnosis is uncertain, a qualified hematologist or oncologist should be consulted

Correct Answer. a

Copyright 2014 Delhi Academy of Medical Sciences, All Rights Reserved. 17/85
(40). In Small cell Ca lung, DNA of the necrotic cells get deposited on vessels known as:

a. Psammoma body

b. Azzopardy effect

c. Button collar lesion

d. Necrotic diathesis

Solution. Ans-40: (b) Azzopardy effect


Ref.: Read the text below
Sol :
Azzopardi phenomenon : In necrotic areas of tumor, dark blue DNA liberated from dead cells sometimes coats vessel walls.
The tumor cells surrounding the vessels show coagulaive necrosis.

Correct Answer. b

Copyright 2014 Delhi Academy of Medical Sciences, All Rights Reserved. 18/85
(41). A young man back from leisure trip has swollen knee joints & foreign body sensation in eyes. Most likely cause is :

a. Reiters syndrome

b. Sarcoidosis

c. Behcets disease

d. SLE

Solution. Ans-41: (a) Reiters syndrome


Ref: Fitzptricks 5th ed
Sol:
C. trachomatis (serovars AK) commonly causes asymptomatic and symptomatic mucosal infections, as well invasive disease
[lymphogranuloma venereum (LGV), hemorrhagic protocolitis].
These infectious syndromes resemble and must be differentiated from those caused by gonococci.
Epidemiology and Etiology
Etiology
C. trachomatis, obligate intracellular bacteria. Major outer-membrane protein delineates >20 serovars (immunotypes):
Trachoma Serovars A, B, Ba, and C.
Mucosal STIs Serovars DK (most common bacterial STIs).
Invasive STIs Serovars L1, L2, L3 (in United States, L2 most commonly)
Age of Onset
Late teens, early twenties, while most sexually active.
Incidence
Prevalence in young American males: 3 to 5% in general medical settings or urban high schools; >10% in asymptomatic soldiers; 15 to 20% in
heterosexual men in STD clinics.
Chlamydial urethritis more common in heterosexual men and high socioeconomic status; gonococcal urethritis more common in homosexual
men and indigent populations.
Prevalence of cervical infection in the United States: 5% for asymptomatic college students; >10% in family planning clinics: >20% in STD
clinics. LGV more common in homosexual men; persons returning from abroad (travelers, sailors, military personnel).
Transmission
Sexual: C. trachomatis in purulent exudate is inoculated onto skin or mucosa of sexual partner and gains entry through minute lacerations
and abrasions. Perinatal.
Syndromes
Nongonococcal (NGU) and postgonococcal urethritis 20 to 40% of NGU in heterosexual men are chlamydial. Also caused by U. urealyticum, T.
vaginalis, HSV.
Epididymitis Most common cause (70%) in sexually active men <35 years.
Reiter syndrome C. trachomatis recovered from urethra in up to 70% of men with untreated nondiarrheal Reiter syndrome and associated
urethritis. Chlamydial and other mucosal infections (Salmonella, Shigella, Campylobacter) thought to initiate aberrant, hyperactive immune
response that produces inflammation at involved target organs in genetically (HA-B27 phenotype) predisposed individuals.
Perinatal infections 50 to 75% of newborns exposed to C. trachomatis at birth acquire infection. 50% of those infected develop clinical
evidence of inclusion conjunctivitis; pneumonitis, otitis media also occur.

Correct Answer. a

Copyright 2014 Delhi Academy of Medical Sciences, All Rights Reserved. 19/85
(42). Mesangial cell contraction is brought about by which of the following chemical mediator?

a. Histamine

b. Bradykinin

c. Nitric oxide

d. PAF

Solution. Ans-42: (d) PAF


Ref.: Read the text below
Sol :
Renal effects and mesangial cell contraction induced by endothelin are mediated by PAF. The recently discovered vasoactive peptide
endothelin and platelet activating factor (PAF) share similar renal effects.
PAF may be a mediator of the effects of endothelin on renal function and glomerular and mesangial cell contraction.

Correct Answer. d

(43). The patients having acute cardiac failure do not show edema, because:

a. The plasma oncotic pressure is high

b. There is renal compensation

c. Increased cardiac output

d. There is a fall in the systemic capillary hydrostatic pressure

Solution. Ans-43: (d) There is a fall in the systemic capillary hydrostatic pressure
Ref: Read the text below
Sol :
Types of Heart Failure
Acute and Chronic Heart Failure
Acute heart failure develops suddenly. The sudden reduction in cardiac output results in systemic hypotension without peripheral
edema. Best examples are acute myocardial infarction and rupture of cardiac valve.
Chronic heart failure develops gradually. Here systemic arterial pressure is well maintained, but edema accumulates. Best examples are
dilated cardiomyopathy and multi- valvular disease.
Compensated heart failure implies that the compensatory changes have prevented the development of overt heart failure. A minor insult
like an infection may precipitate severe heart failure.

Correct Answer. d

(44). Type of pulse in atrial fibrillation.

a. Regularly regular

b. Irregularly regular

c. Regularly irregular

d. Irregularly irregular

Solution. Ans-44: (d) Irregularly irregular


Ref: Read the text below
Sol :
Atrial flutter :
o Fast atrial rate of 220-350/min. with ventricular rate half or one-fourth of the atrial rate
o P waves replaced by flutter waves
o Ventricular rhythm usually regular, unless there is a changing A V block.
Atrial fibrillation :
o Irregularly irregular rhythm
o P waves replaced by fibrillation waves
o Normal QRS complexes.

Correct Answer. d

Copyright 2014 Delhi Academy of Medical Sciences, All Rights Reserved. 20/85
(45). Dissecting aneurysm is a feature of :

a. Blooms syndrome

b. Marfans syndrome

c. Ehlers Danlos syndrome

d. None of the above

Solution. Ans-45: (c) Ehlers Danlos syndrome


Ref: Read the text below
Sol :
The factors that predispose to aortic dissection include :
o Systemic hypertension (a coexisting condition in 70% of patients).
o Cystic medial necrosis.
o Marfan syndrome.
o Ehlers Danlos syndrome.
o Inflammatory aortitis (.e., Takayasus arteritis, giant cell arteritis).
o Congenital aortic valve anomalies (e.g. bicuspid valve).
o Coarctation of the aorta, and
o In otherwise normal women during the third trimester of pregnancy.
Most common cause of aortic dissection : Cystic medial necrosis.
Most common cause of aortic aneurysm : Atherosclerosis.

Correct Answer. c

(46). FISH Technique in genetics is used for :

a. Detection of microdeletion

b. Identification of unknown chromosomal material

c. Interphase analysis

d. All of the above

Solution. Ans-46: (d) All of the above


Ref: Read the text below
Sol :
FISH (fluorescence in situ hybridization) is a cytogenetic technique developed by biomedical researchers in the early 1980s that is used
to detect and localize the presence or absence of specific DNA sequences on chromosomes.
FISH uses fluorescent probes that bind to only those parts of the chromosome with which they show a high degree of sequence
complementarity. Fluorescence microscopy can be used to find out where the fluorescent probe is bound to the chromosomes.
FISH is often used for finding specific features in DNA for use in genetic counseling, medicine, and species identification.
FISH can also be used to detect and localize specific RNA targets (mRNA, lncRNA and miRNA) in cells, circulating tumor cells, and
tissue samples.
In this context, it can help define the spatial-temporal patterns of gene expression within cells and tissues

Correct Answer. d

Copyright 2014 Delhi Academy of Medical Sciences, All Rights Reserved. 21/85
(47). Drug of choice of hyperkalemia :

a. 10% KCL/IV

b. 10% Calcium gluconate/IV

c. 10% NaHCO3/IV

d. Ringer lactate

Solution. Ans-47: (b) 10% Calcium gluconate/IV


Ref: Read the text below
Sol :
CORRECTION OF HYPERKALEMIA
Protect mycocardium
10 mL of 10% calcium gluconate given in the presence of ECG change.
Effect is temporary but dose can be repeated.
Drive K+ into cells :
o Insulin 10 units + 50 mL of 50% glucose followed by regular checks of blood glucose and plasma K+.
o Repeat as necessary.
o Correction of severe acidosis (pH < 6.9) NaHCO3 (1.26%).
o Salbutamol 0.5 mg in 100 ml of 5% glucose over 15 min (rarely used.)
Later :
Deplete body K+ (to decrease plasma K+ over next 24 hrs) polystyrene sulphonate resins :
o 15 orally upto three times daily with laxatives.
o 30 g rectally followed 3-6 hours later by an enema.
o Haemodialysis or peritoneal dialysis.

Correct Answer. b

(48). In pseudo-precoccous puberty there is :

a. Virulization + /Spermatogenesis

b. Spermatogenesis + /Virulization +

c. Spermatogenesis + /Virulization

d. Spermatogenesis - /Virulization

Solution. Ans-48: (a) Virulization + /Spermatogenesis


Ref: Read the text below
Sol :
Precocious pseudopuberty or incomplete isosexual precocity refers to virilization unaccompanied by spermatogenesis.
This distinction is blurred in practice, because pure virilizing syndromes may cause activation of gonadotropin secretion secondarily and
thus be followed by development of spermatogenesis.

Correct Answer. a

Copyright 2014 Delhi Academy of Medical Sciences, All Rights Reserved. 22/85
(49). In vitro DNA amplification is done by :

a. Blotting technique

b. Recombinant technique

c. Electrophoresis

d. Polymerase chain reaction

Solution. Ans-49: (d) Polymerase chain reaction


Ref: Read the text below
Sol :
PCR (in vitro DNA amplification)
Described as being to genes what Gutenberg's printing press was to the written word, PCR can amplify a desired DNA sequence of any
origin (virus, bacteria, plant, or human) hundreds of millions of times in a matter of hours, a task that would have required several days
with recombinant technology.
PCR is especially valuable because the reaction is highly specific, easily automated, and capable of amplifying minute amounts of
sample. For these reasons, PCR has also had a major impact on clinical medicine, genetic disease diagnostics, forensic science, and
evolutionary biology
PCR is a process based on a specialized polymerase enzyme, which can synthesize a complementary strand to a given DNA strand in a
mixture containing the 4 DNA bases and 2 DNA fragments (primers, each about 20 bases long) flanking the target sequence. The mixture
is heated to separate the strands of double- stranded DNA containing the target sequence and then cooled to allow (1) the primers to find
and bind to their complementary sequences on the separated strands and (2) the polymerase to extend the primers into new
complementary strands.
Repeated heating and cooling cycles multiply the target DNA exponentially, since each new double strand separates to become two
templates for further synthesis. In about 1 hour, 20 PCR cycles can amplify the target by a millionfold.

Correct Answer. d

(50). Which of the following drugs is useful in treatment of PSVT?

a. Adenosine

b. Amiodarone

c. Verapamil

d. Digitoxin

Solution. Ans-50: (a) Adenosine


Ref: Read the text below
Sol :
Adenosine, an ultra short acting AV nodal blocking agent, is indicated if vagal maneuvers are not effective.
If successful, followup therapy with diltiazem, verapamil or metoprolol may be indicated. Adenosine may be safely used during
pregnancy.
SVT that does not involve the AV node may respond to other anti-arrhythmic drugs such as sotalol or amiodarone.

Correct Answer. a

Copyright 2014 Delhi Academy of Medical Sciences, All Rights Reserved. 23/85
(51). Decrease vasomotor tone & increase pooling of blood seen in shock due to?

a. Neurogenic shock

b. Pulmonary embolism

c. Cardiac shock

d. Hypovolemia

Solution. Ans-51: (a) Neurogenic shock


Ref.: Read the text below
Sol :
Interruption of sympathetic vasomotor input after a high cervical spinal cord injury, inadvertent cephalad migration or spinal anesthesia,
or devastating head injury may result in neurogenic shock.
In addition to arteriolar dilatation, venodilation causes pooling in the venous system, which decreases venous return and cardiac output.
The extremities are often warm, in contrast to the usual vasoconstriction-induced coolness in hypovolemic or cardiogenic shock.

Correct Answer. a

(52). Acute hyperkalemia is associated with which of the following electrocardiographic changes?

a. QRS widening

b. Prolongation of the ST segment

c. A decrease in the PR interval

d. Prominent U waves

Solution. Ans 52: (a) QRS widening


Ref Read the text below
Sol:
Hyperkalemia leads to partial depolarization of cardiac cells. As a result, there is slowing of the upstroke of the action potential as well
as reduced duration of repolarization.
The T wave becomes peaked, the RS complex widens and may merge with the T wave (giving a sine-wave appearance), and the P wave
becomes shallow or disappears.
Prominent U waves are associated with hypokalemia;ST-segment prolongation is associated with hypocalcemia.

Correct Answer. a

(53). All of the following clinical findings are consistent with severe mitral stenosis except

a. Atrial fibrillation

b. Opening snap late after S2

c. Pulmonary vascular congestion

d. Pulsatile liver

Solution. Ans 53: (b) Opening snap late after S2


Ref Read the text below
Sol:
The time interval between closure of the aortic valve (A2) and the opening snap of mitral stenosis is inversely related to the severity of
mitral stenosis.
In severe mitral stenosis, the left atrial pressure is high. In a patient with elevated left atrial pressures, the mitral valve opens quickly
after closure of the aortic valve (A2) due to the relatively low pressure gradient across the mitral valve in early diastole.
If the left atrial pressure were lower, it would take longer for the pressure gradient across the mitral valve to cause mitral valve
opening.
A short interval between A2 and the opening snap indicates very elevated left atrial pressures.
Atrial fibrillation, pulmonary vascular congestion,pulmonary hypertension, and right-ventricular failure (elevated jugular
pressure,pulsatile liver, peripheral edema) are all potential sequelae of severe mitral stenosis

Correct Answer. b

Copyright 2014 Delhi Academy of Medical Sciences, All Rights Reserved. 24/85
(54). A 55 years old male patient underwent cholecystectomy for gall stone calculus. During surgery the patients pulse was irregularly
irregular, 160/min, BP = 80/50 mm of Hg, temp, 40C. On examination a swelling in the neck was found. Most likely diagnosis is :

a. Thyroid storm

b. Myocardial infarction

c. Pancreatitis

d. SVT

Solution. Ans-54: (a) Thyroid storm


Ref: Read the text below
Sol :
Thyroid storm, a rarely seen disorder, is an extreme form of thyrotoxicosis.
It generally occurs in stressful illnesses, thyroid surgery or RAI administration.
It is manifested by marked delirium, severe tachycardia, vomiting, diarrhea, dehydration and high fever.
The mortality rate in these cases is very high

Correct Answer. a

(55). In patients with hypertrophic cardiomyopathy maximum mutations are found in which gene:

a. b - myosin heavy chain

b. Elastin

c. a tropomyosin

d. Troponin T

Solution. Ans-55: (a) b - myosin heavy chain


Ref: Read the text below
Sol :
Mutations in gene for b - Mysoin heavy chain are associated with 40% of the families with hypertrophic cardiomyopathy.
Troponin T mutations - 15% of the families
- tropomyosin mutations ~5% of the families

Correct Answer. a

(56). Giant "a" waves in the jugular vein can occur all of following except:

a. Aortic stenosis.

b. Pulmonary hypertension.

c. Aortic regurgitation.

d. Tricuspid stenosis.

Solution. Ans-56: (c) Aortic regurgitation.


Ref: Read the text below
Sol :
"a" waves are the positive deflection in the jugular venous pulse following right atrial contraction.
This becomes giant in situations where the atrium is hypertrophied or contracts against resistance, such as constrictive pericarditis,
pulmonary hypertension, and tricuspid stenosis or atresia

Correct Answer. c

Copyright 2014 Delhi Academy of Medical Sciences, All Rights Reserved. 25/85
(57). Most common cause for community acquired pneumonia :

a. Staphylococcus

b. H influenza

c. Pseudomonas

d. Streptococcus

Solution. Ans-57 : (d) Streptococcus


Ref: Read the text below
Sol :
Microbila Causes of Community Acquired Pneumonia by Site of Care

Correct Answer. d

(58). Treatment for childhood hypothyroidism is with :

a. T4

b. T3

c. Levothyroxine

d. TSH

Solution. Ans-58: (c) Levothyroxine


Ref: Read the text below
Sol :
Management of childhood hypothyroidism
The aim of treatment is early detection and early thyroid hormone replacement to ensure that infants do not develop irreversible
neurological disability.
Thyroxine hormone replacement with L-thyroxine is given once daily and titrated to TFTs.
There is no evidence at present to suggest that higher starting doses of thyroxine have more beneficial effect on outcome compared with
standard doses.
TFTs need to be monitored on a regular basis. The frequency of blood tests can be reduced after the first 2 years of life once adequate
replacement is achieved.
T4 should ideally be kept in the upper half of the normal range.
Transient hypothyroidism need not be treated unless the low T4 and raised TSH persist beyond 2 weeks. Treatment is usually
terminated after 3 to 5 months

Correct Answer. c

Copyright 2014 Delhi Academy of Medical Sciences, All Rights Reserved. 26/85
(59). Ideal investigation for adrenal insufficiency is :

a. ACTH stimulation

b. Cortisol level estimation

c. Dexamethasone test

d. Urinary steroid estimation

Solution. Ans-59: (a) ACTH stimulation


Ref: Read the text below
Sol :
ACTH Stimulation test : This is the most specific test for diagnosing adrenal insufficiency. Blood cortisol levels are measured before and
after a synthetic form of adrenocorticotrophic hormone (ACTH), a hormone secreted from the anterior pituitary, is given by injection.

Correct Answer. a

(60). A 67-year-old lady completed a week-long course of antibiotics for a hospital-acquired pneumonia.She is still suffering with a productive
cough with foul-smelling sputum. On further questioning she admits chest pain and occasional haemoptysis. Chest X-ray show a walled
cavity. Which of the following is the most likely diagnosis?

a. Pleural effusion

b. Pulmonary embolus.

c. Recurrent infection

d. Lung abscess.

Solution. Ans 60: (d) Lung abscess


Ref Read the text below
Sol:
This clinical picture points towards a lung abscess. Most frequently, the lung abscess arises as a complication of aspiration pneumonia
caused by mouth anaerobes. Abscesses can be primary or secondary in origin.
A primary abscess is infectious in origin, caused by aspiration or pneumonia in the healthy host whilst a secondary abscess is caused by
an underlying condition (e.g. obstruction), spread from an distal site, bronchiectasis and/ or an immunocompromised state.
It is possible to classify abscesses by the infectious organism, e.g. Staphylococcus lung abscess and anaerobic or Aspergillus lung
abscess.
In this clinical scenario the important factors to identify are the swinging fever and the recurrence of cough with foul smelling sputum
shortly after a chest infection. These are both factors that should raise suspicion of an abscess.
In addition, the X-ray findings of a walled cavity should raise the possibility of the diagnosis.
Empyema is different as it is inflammatory fluid and debris within the pleural space. It may result from an untreated pleural space
infection which subsequently progresses collection in the pleural space.

Correct Answer. d

(61). All the following drugs are associated with an increased risk of osteoporosis in adults except

a. Cyclosporine

b. Dilantin

c. Prednisone

d. Ranitidine

Solution. Ans 61: (d) Ranitidine


Ref Read the text below
Sol:
Multiple drugs are associated with an increased risk of osteoporosis.
In addition to those listed, other anticonvulsants, cytotoxic drugs, excessive thyroxine, aluminum, gonadotropin-releasing hormone
agonists, and lithium are associated with decreased bone mass and osteoporosis.
Histamine antagonists are not associated with osteoporosis.

Correct Answer. d

Copyright 2014 Delhi Academy of Medical Sciences, All Rights Reserved. 27/85
(62). An X-linked recessive disease is characterized by the following :-

a. Vertical transmission

b. 50% female carriers if male is affected and female is normal

c. 50% male affected if female is carrier and male is normal

d. 50% male carriers if female is affected and male is normal

Solution. Ans 62: (c) 50% male affected if female is carrier and male is normal
Ref Read the text below
Sol:
In X-linked recessive disease
Only males are affected.
Unaffected female carriers transmit the disease.
50% of carrier female's offspring inherit mutation-males are affected and females are carriers.
Affected males cannot transmit the disease to their sons but 100% of their daughters are carriers.

Correct Answer. c

(63). The type of astrocytoma which is associated with tuberous sclerosis is

a. Glioblastoma multiforme

b. Gemistocytic astrocytoma

c. Sub ependymal giant cell astrocytoma

d. Pleomorphic xanthoastrocytoma

Solution. Ans-63: (c) Sub ependymal giant cell astrocytoma


Ref: Read the text below
Sol :
Subependymal giant cell astrocytoma occurs in 10-15% of cases of tuberous sclerosis.
(Diffuse) Astrocytoma.
Astrocytoma (low grade or benign astrocytoma)
Anaplastic astrocytoma.
Protoplasmic astrocytoma.
Gemistocytic astrocytoma
Glioblastoma multiforme
Circumscribed astrocytomas
Pilocytic astrocytoma
Pleomorphic xanthoastrocytoma
Subependymal giant cell astrocytom

Correct Answer. c

Copyright 2014 Delhi Academy of Medical Sciences, All Rights Reserved. 28/85
(64). A patient suffers from a congenital deficiency of factor XIII. What would analysis of his blood reveal?

a. Prolonged prothrombin time

b. Prolonged whole blood clotting time

c. Prolonged partial thromboplastin time

d. Easily breakable clot

Solution. Ans-64: (d) Easily breakable clot


Ref: Read the text below
Sol :
Fibrin monomers polymerize to form a coat.
A strong clot requires the presence of fibrin-stabilizing factor (factor XIII), which is released from platelets within the clot.
The other clotting tests determine the activation of extrinsic and intrinsic pathway or the number of platelets

Correct Answer. d

(65). In chronic poisoning with carbon monoxide, CNS changes are particularly marked in :-

a. Cerebral cortex

b. Basal ganglia

c. Cerebellum

d. Brain stem

Solution. Ans-65: (b) Basal ganglia


Ref: Read the text below
Sol :
The slowly developing hypoxia in CO poisoning can insidiously evoke widespread ischemic changes in the CNS ; these are particularly
marked in the basal ganglia & lenticular nuclei.

Correct Answer. b

(66). Hereditary spherocytosis is associated with all except:-

a. Ankyrin

b. Anion exchanger

c. Paladin

d. Glycophorin C

Solution. Ans-66: (d) Glycophorin C


Ref: Read the text below
Sol :
Hemolytic anaemia are associated with defects of red cell membrane proteins.
Hereditary spherocytosis is associated with defects in spectrin, ankyrin, anion exchanger & pallidin.
Hereditary elliptocytosis is associated with spectrin glycophorin C & protein 4.1.
Congenital dyserythropoietic anaemia type 2 is associated with glycophorin A and anion exchanger.

Correct Answer. d

Copyright 2014 Delhi Academy of Medical Sciences, All Rights Reserved. 29/85
(67). Hemophilus species causing endocarditis is :

a. H. influenzae

b. H. aprophilus

c. H. hemolyticus

d. H. aegipticus

Solution. Ans-67: (b) H. aprophilus


Ref: Read the text below
Sol :
Endocarditis causing species of Hemophilus are:
H. aphrophilus
H. paraphrophilus
H. parainfluenzae

Correct Answer. b

(68). Which type of Intracerebral bleed has a good surgical prognosis :-

a. Lobar hemorrhage

b. Putaminal hemorrhage

c. Cerebellar hemorrhage

d. Thalamic bleed

Solution. Ans 68: (c) Cerebellar hemorrhage


Ref Read the text below
Sol:
Surgical evacuation is helpful only in cerebellar bleed.
Ans 69: (a) Rickettsia prowazekii
Ref Read the text below
Sol:
Epidemic typhus due to infection with R.Prowazekii is transmitted by the human body louse (Pediculus humanus corporis), which lives
on clothes and is found in poor hygienic conditions (especially in jails, where the disease it causes is called Jail fever) and usually in cold
areas.

Correct Answer. c

(69). Jail fever is related with :-

a. Rickettsia prowazekii

b. Rickettsia conorii

c. Rickettsia Akari

d. Rickettsia Typhi

Solution. Ans 69: (a) Rickettsia prowazekii


Ref Read the text below
Sol:
Epidemic typhus due to infection with R.Prowazekii is transmitted by the human body louse (Pediculus humanus corporis), which lives
on clothes and is found in poor hygienic conditions (especially in jails, where the disease it causes is called Jail fever) and usually in cold
areas.

Correct Answer. a

Copyright 2014 Delhi Academy of Medical Sciences, All Rights Reserved. 30/85
(70). The Diabetes Control and Complications Trial (DCCT) provided definitive proof that reduction in chronic hyperglycemia

a. Improves microvascular complications in type 1 diabetes mellitus

b. Improves macrovascular complications in type 1 diabetes mellitus

c. Improves microvascular complications in type 2 diabetes mellitus

d. Improves macrovascular complications in type 2 diabetes mellitus

Solution. Ans 70: (a) Improves microvascular complications in type 1 diabetes mellitus
Ref Read the text below
Sol:
The DCCT found definitive proof that reduction in chronic hyperglycemia can prevent many of the complications of type 1 DM. This
multicenter randomized trial enrolled over 1400 patients with type 1 DM to either intensive or conventional diabetes management and
prospectively evaluated the development of retinopathy, nephropathy, and neuropathy.
The intensive group received multiple administrations of insulin daily along with education and psychological counseling. The intensive
group achieved a mean hemoglobin A1C of 7.3% versus 9.1% in the conventional group.Improvement in glycemic control resulted in a
47% reduction in retinopathy, a 54% reduction in nephropathy, and a 60% reduction in neuropathy. There was a nonsignificant trend
toward improvement in macrovascular complications.
The results of the DCCT showed that individuals in the intensive group would attain up to 7 more years of intact vision and up to 5 more
years free from lower limb amputation.
Later, the United Kingdom Prospective Diabetes Study (UKPDS) studied over 5000 individuals with type 2 DM.
Individuals receiving intensive glycemic control had a reduction in microvascular events but no significant change in macrovascular
complications. These two trials were pivotal in showing a benefit of glycemic control in reducing microvascular complications in patients
with type 1 and type 2 DM, respectively.
Another result from the UKPDS was that strict blood pressure control resulted in an improvement in macrovascular complications.

Correct Answer. a

(71). Measurement of which of the following will distinguish primary from secondary hypogonadism?

a. Aldosterone

b. Cortisol

c. Estradiol

d. Luteinizing hormone

Solution. Ans 71: (d) Luteinizing hormone


Ref Read the text below
Sol:
Measurement of luteinizing hormone (LH) or follicle stimulating hormone (FSH) will distinguish primary from secondary hypogonadism
in men with reduced serum testosterone levels.
Elevations in LH and FSH suggest primary gonadal dysfunction, whereas normal or reduced LH and FSH suggest a central hypothalamic
pituitary defect.
Patients with chronic illness such as HIV, end-stage renal disease,COPD,and cancer and patients receiving chronic corticosteroids have
a high frequency of hypogonadism that is associated with muscle wasting.
There are some reports of reversal of hypogonadism in patients with end-stage renal disease on hemodialysis after a renal transplant.

Correct Answer. d

Copyright 2014 Delhi Academy of Medical Sciences, All Rights Reserved. 31/85
(72). Which of the following is not associated with hyponatraemia and hyperkalaemia?

a. Acute hypoadrenalism

b. Carbenoxolone therapy

c. Co-Amilofruse therapy

d. Congestive cardiac failure.

Solution. Ans 72: (b) Carbenoxolone therapy


Ref Read the text below
Sol:
Carbenoxolone therapy may be associated with hypokalaemia and salt retention due to pseudohypoaldosteronism through inhibition of
the enzyme 11 beta Hydroxysteroid dehydrogenase.
Type IV renal tubular acidosis is associated with hyporeninaemic hypoaldosteronism and both hyponatraemia and hyperkalaemia can
occur.
Hypoadrenalism is associated with hyperkalaemia and hyponatraemia as is Cardiac failure, hepatic and renal failure.
Co-amilofruse the combination of amiloride and frusemide may also produce this biochemical picture.

Correct Answer. b

(73). Which of the following agents does not cause photosensitivity eruptions?

a. Furosemide

b. Nickel

c. Psoralens

d. Thiazides

Solution. Ans 73: (b) Nickel.


Ref Read the text below
Sol:
Psoralens, such as 5-methoxypsoralen, are furocoumarins, as are compounds found in limes, figs, celery, and parsnips.
Topical psoralens have been used to treat vitiligo. Ultraviolet light initiates the eruption, only at sites where the topical photosensitizer
contacted the skin. Generally, the eruption resembles a localized sunburn.
Furosemide (a sulfonamide diuretic), chlorpromazine, thiazides, tetracyclines, and sulfonamides all cause generalized photosensitivity
eruptions in sun-exposed surfaces.
Areas of the body less exposed to sunlight are spared, including the eyelid creases and submental anterior neck.
Treatment involves topical corticosteroids, sun avoidance, and discontinuation of the agent.
Nickel exposure can cause a contact dermatitis but not a photosensitivity reaction.

Correct Answer. b

Copyright 2014 Delhi Academy of Medical Sciences, All Rights Reserved. 32/85
(74). Drug of choice for thromboembolism is :

a. Streptokinase

b. Low molecular weight heparin

c. Urokinase

d. Warfarin

Solution. Ans-74: (b) Low molecular weight heparin


Ref: Read the text below
Sol :
Management
Anticoagulant medications include the following:
Heparin or a low-molecular-weight heparin (LMWH)
Subsequent administration of an oral coumarin derivative (typically, warfarin sodium)
Oral factor Xa inhibitors (eg, rivaroxaban)
Thrombolytic options (for initial treatment of patients with acute, massive PE causing hemodynamic instability) include the following:
Tissue plasminogen activator (t-PA; first-choice thrombolytic agent), including the recombinant agents alteplase, reteplase, and
tenecteplase
Streptokinase (risk of antibody development)
Urokinase (of limited availability)
Surgical interventions include the following:
Thrombectomy
Embolectomy (limited to massive PE when thrombolysis is contraindicated or other treatments have failed)
Venous interruption (currently rare)

Correct Answer. b

(75). Neurological and cardiac manifestation of syphilis are seen in :

a. Primary syphilis

b. Secondary syphilis

c. Tertiary syphilis

d. Quaternary syphilis

Solution. Ans-75: (c) Tertiary syphilis


Ref: Read the text below
Sol :
o Classification and clinical features of syphilis

Correct Answer. c

Copyright 2014 Delhi Academy of Medical Sciences, All Rights Reserved. 33/85
(76). Morning dip in peak expiratory flows is a feature of :

a. Nocturnal asthma

b. Emphysema

c. Cardiac asthma

d. Bronchiectasis

Solution. Ans-76: (a) Nocturnal asthma


Ref: Read the text below
Sol :
Validation of morning dip of peak expiratory flow as an indicator of the severity of nocturnal asthma.
Overnight falls in peak expiratory flow (PEF) (with the morning dip of the index) may be considered the hallmark of nocturnal asthma

Correct Answer. a

(77). Pulmonary embolism may present as :

a. Blood tinged sputum

b. Acute breathlessness

c. Chest pain

d. All the above

Solution. Ans-77: (d) All the above


Ref: Read the text below
Sol :
Acute Pulmonary Embolism
History : A characteristic clinical setting, such as prolonged immobilization, recent surgery, congestive heart failure, or recent trauma
will be present.
Oral contraceptives, sickle cell anaemia and polycythaemia are associated with increased incidence of pulmonary embolism.
The usual history is acute onset breathlessness and chest pain, occasionally associated with blood tinged sputum.

Correct Answer. d

(78) Causes of pleural effusion are all except:


.

a. Para pneumonic effusion

b. Pneumocystis carinii

c. TB

d. Meigs syndrome

Solution. Ans-78: (b) Pneumocystis carinii


Ref: Read the text below
Sol :
CAUSES OF PLEURAL EFFUSION

Correct Answer. b

Copyright 2014 Delhi Academy of Medical Sciences, All Rights Reserved. 34/85
(79). Genetic linkage associated with migraine is attributed to defect in :-

a. Sodium channel

b. Calcium channel

c. Potassium channel

d. Acetyl choline receptor

Solution. Ans-79: (b) Calcium channel


Ref: Read the text below
Sol:
A mutation in P/Q type calcium channel on chromosome 19 is associated with familial migraine.
Ans-80: (c) Facial migraine
Ref: Read the text below
Sol:
He has facial migraine or Carotidynia. Patients often have history of dental extraction.
Normal blood test rule out temporal arteritis.
Cluster headache has autonomic features.
Cervical spondylosis causes no jaw pain and is not associated with carotid artery tenderness.

Correct Answer. b

(80). A 60 year old man came with periodic jaw and neck pain with ipsilateral carotid artery tenderness.He had history of dental extraction 1
month back. All his blood counts are normal. He has:-

a. Temporal arteritis

b. Cluster headache

c. Facial migraine

d. Cervical spondylosis

Solution. Ans-80: (c) Facial migraine


Ref: Read the text below
Sol:
He has facial migraine or Carotidynia. Patients often have history of dental extraction.
Normal blood test rule out temporal arteritis.
Cluster headache has autonomic features.
Cervical spondylosis causes no jaw pain and is not associated with carotid artery tenderness.

Correct Answer. c

(81). A 38 year old male patient started developing rapidly progressive dementia with myoclonus. His lumbar puncture was done and it
revealed 14-3-3 protein on immunophoresis. He has :-

a. HIV encephalitis

b. CJ Disease

c. Subacute Sclerosing Pan Encephalitis

d. Kuru

Solution. Ans-81: (b) CJ Disease


Ref: Read the text below
Sol:
Protein 14-3-3 in CSF in virtually diagnostic of CJ Disease. The disease is characterized by rapidly progressive dementia with myoclonus.
14-3-3 elevation also found in herpes simplex encephalitis multiinfarct dementia & stroke.

Correct Answer. b

Copyright 2014 Delhi Academy of Medical Sciences, All Rights Reserved. 35/85
(82). Patients with chronic hepatitis-B are not candidates for Interferon therapy, if they have:

a. Raised ALT - >2 times Normal

b. Detectable markers of HBV replication

c. Decompensated liver disease

d. HbeAg negative chronic hepatitis

Solution. Ans-82: (c) Decompensated liver disease


Ref: Read the text below
Sol:
Interferon therapy has not been effective in very young children infected at birth, immunosuppressed persons,
Asian patients with minimal to mild ALT elevations, or in patients with decompensated chronic hepatitis B (in whom such therapy can
actually be detrimental, sometimes precipitating decompensation, often associated with severe adverse effects).

Correct Answer. c

(83). Symptomatic patients diagnosed with mutations in the RyR2-encoded cardiac ryanodine receptor should receive what first line therapy?

a. Calcium channel blocker

b. Beta blocker

c. ICD

d. Surgical myectomy

Solution. Ans-83: (c) ICD


Ref: Read the text below
Sol:
Patients with symptomatic catecholaminergic polymorphic VT should receive an ICD as first line therapy since other therapies, such as
calcium channel and beta blockers, have not been shown to be sufficiently protective.

Correct Answer. c

(84). All of the following are congenital heart defects associated with an increased risk of an accessory pathway except:

a. Shones syndrome

b. Ebsteins anomaly

c. Congenitally corrected transposition of the great vessels

d. HCM

Solution. Ans-84: (a) Shones syndrome


Ref: Read the text below
Sol:
Shones syndrome, which is manifest by multiple LV outflow obstructions, is not associated with an increased risk of an accessory
pathway. All of the other conditions convey an increased risk, in particular Ebsteins anomaly.

Correct Answer. a

Copyright 2014 Delhi Academy of Medical Sciences, All Rights Reserved. 36/85
(85). All the following findings in aortic stenosis suggest severe aortic obstruction or left ventricular decompensation except :-

a. Paradoxic splitting of S2

b. S3 in an adult or S4 in an adolescent

c. Prolonged late - Peaking systolic ejection murmur

d. Ejection sound.

Solution. Ans-85: (d) Ejection sound.


Ref: Read the text below
Sol:
An ejection sound, typically heard in the presence of a bicuspid aortic valve, usually disappears as the aortic
valve becomes calcified and immobile. The presence of an ejection sound provides a clue to the cause (i.e. Bicuspid
aortic valve) but not the severity.
Paradoxic splitting of S2 can be heard especially in younger patients, and is a manifestation of significant aortic
obstruction when it is found.
In older patients as the aortic valve becomes calcified and restricted in motion. the aortic component becomes
inaudible and S2 is single.
An S3 in an adult patient with aortic stenosis implies cardiac decompensation.
An S4 in an adolescent is a manifestation of left ventricular hypertrophy and also implies significant aortic
stenosis in this age group.
A prolonged late peaking systolic ejection murmur is typical for severe aortic stenosis.

Correct Answer. d

(86). According to GOLD criteria stage III COPD is defined as:

a. FEV1/FVC<0.8 and FEV1>30% to<50% predicted

b. FEV1/FVC<0.7 and FEV1>50% to<80% predicted

c. FEV1/FVC<0.7 and FEV1>30% to<50% predicted

d. FEV1/FVC<0.8 and FEV1>50% to<80% predicted

Solution. Ans-86: (c) FEV1/FVC<0.7 and FEV1>30% to<50% predicted


Ref: Read the text below
Sol:

Correct Answer. c

Copyright 2014 Delhi Academy of Medical Sciences, All Rights Reserved. 37/85
(87). Regarding idiopathic Parkinson's disease true is:-

a. There is degeneration primarily of cells of globus pallidus

b. Classical features include tremors, bradykinesia and spasticity

c. There is an associated vertical gaze palsy

d. Anticholinergics are most effective in relieving tremors

Solution. Ans-87: (d) Anticholinergics are most effective in relieving tremors


Ref: Read the text below
Sol:
In the idiopathic Parkinson's disease there is degeneration primarily of cells in the pigmented substantia nigra,
pars compacta and the locus coeruleus in the midbrain.
Parkinson's disease is a syndrome consisting of variable combination of tremor, rigidity (rather than spasticity),
bradykinesia and a characteristic disturbance of gait and posture.
Vertical gaze palsy is characteristic of progressive supranuclear palsy.

Correct Answer. d

(88). A 42-year old man who has had difficulty concentrating on his job lately comes to medical attention because of irregular, jerky
movements of his extremities and figures. His one sister and an uncle died in mental institutions, and his mother became demented in
middle age. The most likely diagnosis is

a. Alcoholic cerebral degeneration

b. Huntington's chorea

c. Wilson's disease

d. Gilles de la Tourette's disease

Solution. Ans-88: (b) Huntington's chorea


Ref: Read the text below
Sol :
Huntington's chorea, which is inherited as an autosomal dominant trait, is characterized by dementia and
choreiform movements.
The motor disorder may include grimacing, respiratory spasms, speech irregularity, and a dancing, jangling
quality in the gait.
Laboratory workup is normal except that atrophy of the caudate nucleus may be seen on a carefully evaluated
CT or MRI scan.
Through the use of DNA linkage analysis, patients can be tested before disease development if this is
appropriate from a psychosocial standpoint.
The disease-specific gene is located on the short arm of chromosome 4.

Correct Answer. b

Copyright 2014 Delhi Academy of Medical Sciences, All Rights Reserved. 38/85
(89). Cardiotoxicity induced by Doxorubicin can be reduced by :-

a. Dexrazoxane

b. Amifostine

c. Leucovorin

d. Bisphosphonates

Solution. Ans-89: (a) Dexrazoxane


Ref: Read the text below
Sol :
Cardiotoxicity induced by Doxorubicin is an important side effect which can be modified by the use of Dexrazoxane.
Dexrazoxane : Iron chelator that decreases free radical formation ~ Protects myocardium from anthracycline induced injury.
Amifostine Radio protective agent (Reduces renal toxicity caused by cisplatin) Leucovorin: Reduces bone marrow & mucosal toxicity
caused by methotrexate other measures [(Vigorous hydration, bicarbonate loading) Asparaginase, Carboxypeptidase G2] Mesna: Reduces
bladder toxicity caused by Cyclophosphamide.
Bisphosphonate: Reduces bone pain & frequency of new skeletal events in metastatic breast cancer & Multiple myeloma.

Correct Answer. a

(90). All are effective in Alzheimer's disease except

a. Rivastigmine

b. Donepezil

c. Piribedil

d. Galantamine

Solution. Ans-90: (c) Piribedil


Ref: Read the text below
Sol :
Piribedil is Dopamine Agonist effective in treatment of Parkinsonism.
Drugs effective in the treatment of Alzheimer's disease are Tacrine, Rivastigmine,
Donepezil, Galantamine, antioxidants (Vitamin E, Vitamin C), glutamate antagonists etc.

Correct Answer. c

(91). All of the following will cause CNS vasculitis; except

a. SLE

b. Whipple's disease

c. Granulomatous vasculitis

d. PAN

Solution. Ans-91: (b) Whipple's disease


Ref: Read the text below
Sol :
CNS vasculitis
Primary - Very rare.
Secondary - Associated with PAN
Granulomatous vasculitis
SLE
Whipple disease may involve CNS but inflammation is usually absent.

Correct Answer. b

Copyright 2014 Delhi Academy of Medical Sciences, All Rights Reserved. 39/85
(92). Which of the following are characteristic of acute pericardial effusion:

a. Giant "a" waves in the jugular venous pulse.

b. Pulsus paradoxus.

c. Right ventricular end-diastolic collapse on echocardiography.

d. All of above

Solution. Ans-92: (d) All of above


Ref: Read the text below
Sol :
Acute pericardial effusion causes cardiac tamponade with
Giant "a" waves in the jugular venous pulse.
Pulsus paradoxus.
Right ventricular end-diastolic collapse on echocardiography.
Low blood pressure.
Cold extremities.
In addition the heart sounds may be muffled and there may be signs of shock

Correct Answer. d

(93). In all patients in whom primary pulmonary hypertension is confirmed ,the next step in the management of the patient is

a. Acute drug testing with short-acting pulmonary vasodilators

b. High-dose nifedipine

c. Intravenous prostacyclin

d. Lung transplantation

Solution. Ans-93: (a) Acute drug testing with short-acting pulmonary vasodilators
Ref: Read the text below
Sol :
In all patients in whom primary pulmonary hypertension is confirmed, acute drug testing with a pulmonary vasodilator is necessary to
assess the extent of pulmonary vascular reactivity.
Inhaled nitric oxide, intravenous adenosine, or intravenous prostacyclin have all been used.
Patients who have a good response to the short-acting vasodilator are tried on a long-acting calcium channel antagonist under direct
hemodynamic monitoring. Prostacyclin has been approved for patients who are functional class III or IV and have not responded to
calcium channel antagonists.
Lung transplantation is reserved for late stages of the disease when patients are unresponsive to prostacyclin. The disease does not
appear to recur after transplantation

Correct Answer. a

Copyright 2014 Delhi Academy of Medical Sciences, All Rights Reserved. 40/85
(94). All are seen in Mitral stenosis except :

a. LVH

b. Left atrial dilation

c. Pulmonary hypertension

d. Atrial fibrillation

Solution. Ans-94: (a) LVH


Ref: Read the text below
Sol :
Mitral stenosis is a condition in which the mitral valve leaflets become thickened and the commissures fused along with thickening and
shortening of the chordate tendinea.
Critical MS occurs when the opening is reduced to 1 sq. cm. At this stage, elevated left atrial pressure (LAP) is required to maintain a
normal cardiac output. This increase in LAP raises pulmonary venous and capillary pressures, resulting in exertional dyspnea. As the
disease progresses, chronic elevation of LAP leads to pulmonary hypertension, tricuspid and pulmonary incompetence and eventual right
heart failure.
Progressive dilation of the left atrium predisposes to two further complications. One is the development of mural thrombi. These thrombi
embolize in 20% of patients.
The other significant complication is the development of atrial fibrillation (Afib) which occurs in upto 40% of patients.
The left ventricular function and size are often normal

Correct Answer. a

(95). All are HUMAN prion diseases except :

a. Chronic Wasting Disease (CWD)

b. Fatal familial insomnia

c. Creutzfeldt-Jakob disease

d. Kuru

Solution. Ans-95: (a) Chronic Wasting Disease (CWD)


Ref: Read the text below
Sol :
Human Prion Diseases
Creutzfeldt-Jakob Disease (CJD).
Variant Creutzfeldt-Jakob Disease (VCJD).
Gerstmann Straussler Scheinker Syndrome.
Fatal Familial Insomnia.
Kuru.

Correct Answer. a

Copyright 2014 Delhi Academy of Medical Sciences, All Rights Reserved. 41/85
(96). All are true about meningococcal meningitis except :

a. DIC

b. Absence of meningeal signs indicates poor prognosis

c. Meningococcemia is associated with petechial rashes

d. Sensitivity to light (photophobia)

Solution. Ans-96: (b) Absence of meningeal signs indicates poor prognosis


Ref: Read the text below
Sol :
Features of Meningoccal Meningitis
Rash, pinpoint red spots
Purple, bruise like areas
High fever
Severe headache
Severe malaise (General ill feeling)
Nausea
Vomiting
Stiff neck
Sensitivity to light (photophobia)
Mental status changes.
Prognosis
The prognosis for meningococcal meningitis is fair if the patient does not have focal neurological deficits and is not stuporous or
comatose.
The prognosis of meningococcal disease is poor when the infection has a septicemia component.

Correct Answer. b

(97). True about acute rheumatic fever :

a. Group A streptococci has been linked to it epidemiologically

b. Usually associated with history of sore throat

c. All are sensitive to penicillin

d. All of the above

Solution. Ans-97: (d) All of the above


Ref: Read the text below
Sol :
Group A Streptococus is thought to cause the myriad of clinical diseases in which the hosts immunologic response to bacterial antigens
cross-react with various target organs in the body, resulting in molecular mimicry. In fact, autoantibodies reactive against the heart have
been found in patients with rheumatic carditis.
The antibody can cross react with brain and cardiac antigens, and immune complexes are present in the serum. The problem has been
the uncertainty of whether these antibodies are the cause or result of myocardial tissue injury.
The clinical syndrome typically follows a streptococcal pharyngitis.
Penicillin is the drug of choice in persons who are not at risk of allergic reaction

Correct Answer. d

Copyright 2014 Delhi Academy of Medical Sciences, All Rights Reserved. 42/85
(98). Metastatic calcification is seen in all, except

a. Hyperparathyroidism

b. Milk alkali syndrome

c. Multiple myeloma

d. Damaged heart valves

Solution. Ans-98: (d) Damaged heart valves


Ref: Read the text below
Sol :
Metastatic calcification occurs in normal tissues whenever there is hypercalcemia.
In contrast, dystrophic calcification occurs in non-viable or dying tissues and is almost inevitable in the atheromas of advanced
atherosclerosis, despite normal serum levels of calcium

Correct Answer. d

(99). Torsades de pointes is characterized by all of the following except:

a. Prolonged QT interval

b. Exacerbation by bradycardia with short-long coupling intervals

c. Polymorphic VT

d. Often provoked during amiodarone administration

Solution. Ans-99: (d) Often provoked during amiodarone administration


Ref: Read the text below
Sol:
Torsades de pointes is characterized by prolonged QT intervals, exacerbation by bradycardia, short-long couple intervals, salvos of
nonsustained polymorphic VT before degeneration into a sustained ventricular arrhythmia, and polymorphic VT with characteristic
twisting around the axis morphology.
Although amiodarone often prolongs the QT interval, it rarely causes torsades de pointes

Correct Answer. d

Copyright 2014 Delhi Academy of Medical Sciences, All Rights Reserved. 43/85
(100). Diagnostic features of allergic bronchopulmonary aspergillosis (ABPA) include all of the following features, except :-

a. Changing pulmonary infiltrates

b. Peripheral eosinophilia

c. Serum precipitins against aspergillus fumigatus

d. Occurrence in patients with old cavitatory lesions

Solution. Ans-100: (d) Occurrence in patients with old cavitatory lesions


Ref: Read the text below
Sol:
Diagnostic features of ABPA
Main diagnostic criteria
Bronchial asthma
Pulmonary infiltrates
Peripheral eosinophilia
Immediate wheal-and-flare response to A.fumigatus
Serum-precipitins to A.fumigatus
Elevated serum IgE
Central bronchiectasis
Other diagnostic features
History of brownish plugs in sputum
Culture of A. fumigatus from sputum
Elevated IgE (and IgG) class antibodies for A.fumigatus
Aspergilloma but not the ABPA occurs in patients with old cavitatory

Correct Answer. d

(101). If patients with acute rheumatic fever receive appropriate antibiotic therapy, what percent develop severe cardiac disease (class IV
rheumatic heart disease) ?

a. 10 percent

b. 5 percent

c. 0.5 percent

d. 1 percent

Solution. Ans-101: (d) 1 percent


Ref: Read the text below
Sol :
If patients with acute rheumatic fever receive appropriate prophylactic antibiotic therapy, about 1 percent develop severe (class IV)
cardiac disease and 4 percent develop debilitating rheumatic heart disease.

Correct Answer. d

Copyright 2014 Delhi Academy of Medical Sciences, All Rights Reserved. 44/85
(102). Hypercalcemia would not be expected to occur as a result of

a. Lung carcinoma

b. Sarcoidosis

c. Hypothyroidism

d. Administration of thiazide drugs

Solution. Ans 102: (c) Hypothyroidism


Reference: Read the text below
Sol:
Hypercalcemia often is associated with malignancies and is due either to skeletal metastasis or to a humoral substance secreted by the
tumor.
Hypercalcemia accompanying sarcoidosis appears to be due to enhanced synthesis of vitamin D.
Thiazide diuretics cause hypercalcemia by decreasing the urinary clearance of calcium. Lithium may cause a mild hypercalcemia by
elevating plasma parathormone levels.
Calcium levels in persons with hypothyroidism are either normal or low as a result of decreased bone turnover.

Correct Answer. c

(103). The most common cause of bacterial meningitis in adults is

a. Streptococcus pneumoniae

b. Neisseria meningitidis

c. Hemophilus influenzae

d. Staphylococcus aureus

Solution. Ans 103: (a) Streptococcus pneumoniae


Reference: Read the text below
Sol:
Streptococcus pneumoniae is responsible for between 30 and 50% of cases of bacterial meningitis in adults, and Neisseria meningitides
between 10 and 35% of cases. Hemophilus influenzae meningitis occurs primarily in children,and group B Streptococcus meningitis
usually affects infants.
Staphylococcus is the most common cause of bacterial meningitis associated with shunting procedures.
Acute otitis media and mastoiditis, recent head injury, sickle cell anemia, chronic alcoholism,immunoglobulin deficiency, and
splenectomy all predispose

Correct Answer. a

(104). ZollingerEllison syndrome is associated with all of the following conditions or laboratory findings except

a. Severe peptic ulcer disease

b. Basal gastric acid hypersecretion

c. Hypergastrinemia

d. Lack of response to secretin stimulation tests

Solution. Ans 104: (d) Lack of response to secretin stimulation tests


Reference: Read the text below
Sol:
The ZollingerEllison syndrome is characterized by severe peptic ulcer disease and, in many patients, diarrhea.
Basal gastric acid hypersecretion and hypergastrinemia are due to the presence of pancreatic gastrinomas,which are nonbeta islet-cell
tumors.
Serum gastrin levels increase markedly following the injection of secretin or gastrointestinal hormone (GIH).
In normal subjects and in persons with duodenal ulcer, serum gastrin levels either do not increase or increase slightly after secretin
stimulation. Most affected individuals can be successfully treated with H2-receptor antagonists, such as cimetidine,at higher dosages
than are used to treat common duodenal ulcers.
An attempt also should be made to remove the gastrinoma

Correct Answer. d

Copyright 2014 Delhi Academy of Medical Sciences, All Rights Reserved. 45/85
(105). The predominant immunoglobulin deposited in the glomeruli and skin of persons with HenochSchnlein purpura is

a. Immunoglobulin A

b. Immunoglobulin D

c. Immunoglobulin E

d. Immunoglobulin G

Solution. Ans 105: (a) Immunoglobulin A


Reference: Read the text below
Sol:
HenochSchnlein purpura is one of the few diseases in which immunoglobulin A (IgA) is the predominant immunoglobulin deposited in
the glomeruli and skin.
In many other glomerulopathies, IgG and IgM are the predominant immunoglobulins deposited in glomeruli.
Henoch-Schnlein purpura generally has a benign course characterized by arthralgias, abdominal pain, and nonthrombocytopenic
purpura in addition to glomerulonephritis.
However, progressive renal failure may occur in a small percentage of patients. Treatment is usually symptomatic.

Correct Answer. a

(106). The best test to determine the earliest signs of depletion of body iron stores is

a. Plasma ferritin

b. Plasma transferrin

c. Red cell free protoporphyrin

d. Bone marrow iron stain

Solution. Ans 106: (d) Bone marrow iron stain


Reference: Read the text below
Sol:
A stain of the bone marrow with Prussian blue stain detects depletion of marrow iron stores, the earliest sign of iron deficiency.
As the extent of iron deficiency increases, plasma ferritin levels fall and subsequently there is a rise in plasma transferrin.
Plasma iron and transferrin saturation fall and free erythrocyte protoporphyrin increases when the amount of available iron declines
below the minimal amount required for erythropoiesis.

Correct Answer. d

(107). The single most important predictor of the outcome of a patient undergoing cardiac surgery is

a. The pulmonary capillary wedge pressure

b. The patients age

c. The ejection fraction

d. Concomitant disease processes

Solution. Ans-107: (c) The ejection fraction


Ref: Read the text below
Sol :
The single most important predictor of outcome in a patient undergoing cardiac surgery is the ejection fraction.

Correct Answer. c

Copyright 2014 Delhi Academy of Medical Sciences, All Rights Reserved. 46/85
(108 All drugs cause interstitial lung disease, except :
).

a. Phenytoin sodium

b. Sulphonamides

c. Busulphan

d. Alpha methyl dopa

Solution. Ans-108: (d) Alpha methyl dopa


Ref.: Harrison 16th/582
Sol :
All drugs cause interstitial lung disease, except Alpha methyl dopa

Correct Answer. d

(109). Thickening of pulmonary membrane is seen in :

a. Asthma

b. Emphysema

c. Bronchitis

d. Skeletal defect

Solution. Ans-109 : (a) Asthma


Ref.: Robbins 7th/726
Sol:

Correct Answer. a

Copyright 2014 Delhi Academy of Medical Sciences, All Rights Reserved. 47/85
(110). Diagnostic features of allergic bronchopulmonary aspergillosis (ABPA) include all of the following except :

a. Changing pulmonary infiltrates

b. Peripheral eosinophilia

c. Serum precipitins against aspergillosis fumigans

d. Occurrence in patients with old cavitatory lesions

Solution. Ans-110: (d) Occurrence in patients with old cavitatory lesions


Ref.: Harrison 17th/1610
Sol :
Allergic bronchopulmonary aspergillosis is pulmonary hypersensitivity disorder caused by allergy to fungal antigens that colonize the
tracheobronchial tree. It most commonly occurs in atopic asthmatic individuals in response to antigen of aspergillus species.

Correct Answer. d

(111). The key factor in the transport of carbon dioxide as bicarbonate is :

a. The high solubility of CO2 in H2O

b. The presence of Hb in blood

c. The presence of carbonic anhydrase in the erythrocytes

d. The acidic nature of carbon dioxide and the alkaline

Solution. Ans-111: (c) The presence of carbonic anhydrase in the erythrocytes


Ref.: Ganong 22nd/669-670
Sol :

Correct Answer. c

Copyright 2014 Delhi Academy of Medical Sciences, All Rights Reserved. 48/85
(112). A two-year-old boy is brought to emergency with severe anemia (Hb = 2 gm%) and features of congestive heart failure. The most
appropriate immediate therapy for this boy would be :

a. Packed cell transfusion

b. Partial exchange transfusion

c. Whole blood transfusion

d. Parenteral iron injection

Solution. Ans-112: (a) Packed cell transfusion


Ref.: Ghai OP. essentials of Pediatrics 6th pg. 301
sol :
Packed cell transfusion indicated only in case o hemorrhage, urgen surgery, and severe anemia with impending congestive heart failure.
When the hemoglobin is <4 gm%. Only packed cell should be given at the rate of 2-3 ml/kg with monitoring and if needed then diuretic
therapy.

Correct Answer. a

(113). What is them most common presentation of intracranial aneurysm?

a. Coarctation of aorta

b. Systemic hypertension

c. Hypotension

d. Intracranial haemorrhage

Solution. Ans-113: (d) Intracranial haemorrhage


Ref.: CSDT- 916
Sol :
Most common type of intracranial aneurysm is saccular aneurysm.
Most aneurysms present with rupture.
Most common nerve involved is oculomotor nerve.
The classic presentation is sudden severe headache that may be associated with nausea vomiting painful nuchal rigidity and
photogphobia

Correct Answer. d

(114). The following biochemical markers are a measure of bone resorption except

a. Serum alkaline phosphatase

b. Serum cross linked N telopeptide

c. Serum cross linked C-telopeptide

d. Urine deoxypyridinoline

Solution. Ans-114: (a) Serum alkaline phosphatase


Ref.: Harrisons Principles of Internal Medicine 17th pg. 2368
Sol :
Osteoblastic activity can be measured by measuring bone specific alkaline phosphatase. Besides it, serum osteocalcin, glycolytic enzyme
and phosphorylase are also markers.
Osteoclastic activity can be assessed by measurement of product of collagen degradation, acid phosphatase, and glycolic acid hydrolase
collagenase.

Correct Answer. a

Copyright 2014 Delhi Academy of Medical Sciences, All Rights Reserved. 49/85
(115). Sequestration lung is best diagnosed by

a. C.T. Scan

b. M. R.I

c. Barium swallow

d. Angiography

Solution. Ans-115: (d) Angiography


Ref.: Sumer Sethi 4th/35
Sol :

Correct Answer. d

(116). All of the following would be appropriate management for chronic arterial insufficiency except

a. Low-fat, low-cholesterol diet

b. Smoking cessation

c. Trental therapy

d. Warfarin therapy

Solution. Ans-116: (d) Warfarin (Coumadin) therapy


Ref: Read the text below
Sol :
Treatment for chronic arterial insufficiency includes reduction of modifiable risk factors for atherosclerosis (smoking, high-fat/hig-
-cholesterol diet, hypertension, and diabetes).
A daily exercise program that includes walking past the point of claudication would enhance the development of collateral circulation.
Trental is a medication that enhances the ability of red cells to pass through stenotic vessels and may improve claudication in some
patients.
Warfarin (Coumadin) is used for long-term management and prevention of recurrent acute arterial insufficiency that is embolic in origin.

Correct Answer. d

(117). Diabetic ketoacidosis (DKA) is most likely to be precipitated by which of the following ?

a. Infection

b. Inadequate food intake

c. Excessive exercise

d. Excessive sulfonylurea intake

Solution. Ans-117: (a) Infection


Ref: Read the text below
Sol :
This is the only choice that is likely to precipitate DKA.
All the other choices are causes of hypoglycemia.

Correct Answer. a

Copyright 2014 Delhi Academy of Medical Sciences, All Rights Reserved. 50/85
(118). Which of the following signs or symptoms of hypoglycemia is due to beta-adrenergic stimulation?

a. Seizures

b. Motor incoordination

c. Sweating

d. Headache

Solution. Ans-118: (c) Sweating


Ref: Read the text below
Sol :
The hypoglycemic patient may complain of sweating. Palpitations, hunger, tremor, nervousness, and weakness. These all result from
stimulation of the beta-adrenergic system (epinephrine).
These signs may be blunted or absent in the patient taking beta blocker, such as propanolol (Inderal).
Patients may have lightheadedness, diplopia, headache, motor incoordination, confusion, obtundation, seizures, and coma.
These result from glucose starvation in the central nervous system.

Correct Answer. c

(119). Which is most important in the treatment of hypoglycemia ?

a. Adequate fluid replacement

b. Intravenous insulin infusion

c. Administration of glucose, preferably intravenously

d. Exercise, to mobilize fats

Solution. Ans-119: (c) Administration of glucose, preferably intravenously


Ref: Read the text below
Sol :
Intravenous glucose (dextrose ) is the treatment for hypoglycemia. In early stages, an alert patient may notice tremulousness, hunger,
seating, and palpitations and be able to take oral glucose or simply food. Absorption is not guaranteed and onset of action may be
delayed.
Thus, intravenous glucose is always best, when available. The comatose patient will awaken quickly if hypoglycemia has not gone so long
that permanent brain damage has occurred.
Even with rapid awakening, the hypoglycemic patient must be carefully monitored. Remember the long half-life of the sulfonylureas.
If these are the cause of the hypoglycemia, repeat attacks may occur after correction, and repeat therapy may be needed.

Correct Answer. c

Copyright 2014 Delhi Academy of Medical Sciences, All Rights Reserved. 51/85
(120). Pulmonary embolism is seen in all except :

a. Fanconis anemia

b. Paroxysmal nocturnal haemoglobinuria

c. Oral contraception

d. Old age

Solution. Ans-120: (a) Fanconis anemia


Ref.: Harrison 17th/1563
Sol :

Correct Answer. a

(121). A patient has been in the cardiac care unit with an acute anterior myocardial infarction. He develops the abnormal rhythm shown below.
You should

a. Give digoxin

b. Consult for pacemaker

c. Perform cardioversion

d. Give propranolol

Solution. Ans-121: (b) Consult for pacemaker


Ref: Read the text below
Sol:
The ECG shows complete heart block.
Although at first glance the P waves and QRS complexes may appear related, on closer inspection they are completely independent of
each other, i.e., dissociated.
Complete heart block in the setting of acute myocardial infarction requires at least temporary, and often permanent, transvenous
pacemaker placement. Atropine may be used as a temporary measure.
You would certainly want to avoid digoxin, beta blockers, or any other medication that promotes bradycardia.
There is no indication on this strip for cardioversion such as for atrial fibrillation/ flutter or ventricular tachycardia/fibrillation. Lidocaine
would be relatively contraindicated in that it might suppress the ventricular pacemaker, leading to asystole.

Correct Answer. b

Copyright 2014 Delhi Academy of Medical Sciences, All Rights Reserved. 52/85
(122). The following drugs are contra-indicated for the treatment of atrial fibrillation associated with the WPW syndrome:

a. Adenosine

b. Amiodarone

c. Verapamil

d. Sotalol

Solution. Ans- 122 : (c) Verapamil


Ref: Read the text below
Sol:
In WPW digoxin or calcium channel blockers may increase anterograde conduction through the bypass tract, so they should be avoided.

Correct Answer. c

(123). A 3 day old baby, discharged following a normal routine postnatal examination, is rushed to hospital with a thready pulse and heart rate
of 180/min, a 5 cm liver edge, and a capillary refill time of 6 seconds. The following cardiac lesions should be considered:

a. Transposition of the great arteries

b. Double outlet right ventricle

c. Pulmonary atresia

d. Hypoplastic left heart syndrome

Solution. Ans- 123 : (d) Hypoplastic left heart syndrome


Ref: Read the text below
Sol:
Neonatal shock may be caused by infection, cardiac lesions or metabolic abnormalities. TGA, double outlet right ventricle, and
pulmonary atresia cause cyanosis.
Hypoplastic left heart syndrome, truncus arteriosus, coarctation, or critical aortic stenosis are the likely cardiac causes of this
presentation.

Correct Answer. d

(124). Diffusion capacity of carbon monoxide:

a. Is a specific measure of lung perfusion.

b. VQ imbalances is directly interfere with carbon dioxide diffusion capacity.

c. Depends on the surface area available for gas exchange.

d. Is increased in cigarette smokers.

Solution. Ans- 124 : (c) Depends on the surface area available for gas exchange.
Ref: Read the text below
Sol:
By Fick's law, the volume of gas diffusing across a membrane equals A/T x D x difference in partial pressure.
In life it is impossible to measure accurately the area (A) or the thickness (T), and these are subsumed into a single constant, the
diffusion capacity for carbon monoxide. DL=volume of transferred carbon dioxide divided by partial pressure difference between the
alveoli and the capillary blood.
Since the capillary blood normally does not contain carbon dioxide this term disappears.
Diffusion will be increased in healthy compared with unhealthy lungs, where the thickness is likely to increase and the surface area
available for gas exchange to decrease.
VQ imbalances can indirectly interfere with carbon dioxide diffusion capacity by decreasing the available area of lung for gas exchange,
but it is not a specific measure of lung perfusion

Correct Answer. c

Copyright 2014 Delhi Academy of Medical Sciences, All Rights Reserved. 53/85
(125). A 29-year-old woman is in the intensive care unit with rhabdomyolysis due to compartment syndrome of the lower extremities after a car
accident. Her clinical course has been complicated by acute renal failure and severe pain. She has undergone fasciotomies and is
admitted to the intensive care unit. An electrocardiogram (ECG) is obtained (shown below). What is the most appropriate course of
action at this point?

a. 18-lead ECG

b. Coronary catheterization

c. Hemodialysis

d. Intravenous fluids and a loop diuretic

Solution. Ans-125: (d) Intravenous fluids and a loop diuretic


Ref: Read the text below
Sol:
This ECG shows a short ST segment that is most prominent in V2, V3, V4, and V5. Hypercalcemia, by shortening the duration of
repolarization, abbreviates the total time from depolarization through repolarization.
This is manifested on the surface ECG by a short QT interval. In this scenario, the hypercalcemia is due to the rhabdomyolysis and renal
failure. Fluids and a loop diuretic are an appropriate therapy for hypercalcemia.
Hemodialysis is seldom indicated. Hemodialysis is indicated for significant hyperkalemia, which may also develop after rhabdomyolysis,
manifest by tenting of the T waves or widening of the QRS.
Classic ECG manifestations of a pulmonary embolus (S1, Q3, T3 pattern) are infrequent in patients with pulmonary embolism (PE),
though the changes may be seen with massive PE.
There are no signs of myocardial ischemia on this ECG, which would make coronary catheterization and 18- lead ECG interpretation of
low yield.

Correct Answer. d

(126). Cluster headache is associated with all, except:

a. Unilateral headache

b. Onset typically in 20-50 years of life

c. Affects predominantly females

d. Commonly awakens the patient from sleep

Solution. Ans-126: (c) Affects predominantly females


Ref: Read the text below.
Sol:
Cluster headache:
Affects predominantly males (7-8 times more than females)
Also known as Raeder's syndrome, histamine cephalalgia, and sphenopalatine neuralgia.
Lateralized
Peak at 30-50 years
Excruciating, deep, explosive pain which begins without warning, reaches it's peak within 5 minutes and lasts
for 45 minutes and commonly awakens the patients from sleep and is associated with homolateral lacrimation and
reddening of eye and ptosis and nasal stuffiness.

Correct Answer. c

Copyright 2014 Delhi Academy of Medical Sciences, All Rights Reserved. 54/85
(127). A 56 year old coal mine worker presented with joint pains, cutaneous nodules and occasional cough with dyspnoea. His chest x-ray
showed multiple small nodules in both lungs with cavitation and calcification. Most likely diagnosis is:

a. Silicosis

b. Caplan's syndrome

c. Sjogren's syndrome

d. Sarocidosis

Solution. Ans-127: (b) Caplan's syndrome


Ref: Read the text below.
Sol:
Caplan's syndrome is a rare condition characterized by the presence of necrobiotic nodules in the periphery of the
lung in coal workers with rheumatoid arthritis (Coal worker's pneumoconiosis + Rheumatoid arthritis)

Correct Answer. b

(128). In a case of first idiopathic venous thromboembolism event; current recommendations for warfarin therapy is for : -

a. Less than 3 months

b. 36 months

c. 6 months

d. 12 months to lifetime

Solution. Ans-128: (c) 6 months


Ref: Read the text below.
Sol:
Current ACCP recommendations for duration of warfarin therapy
Recommendations for Long-Term Anticoagulation

Correct Answer. c

Copyright 2014 Delhi Academy of Medical Sciences, All Rights Reserved. 55/85
(129). In lung perfusion scanning:

a. Emphysema and pulmonary embolism give similar appearances.

b. Iodine sensitivity is a contraindication.

c. May show decreased upper lobe perfusion in mitral stenosis.

d. Shows decreased perfusion in McLeod's Syndrome.

Solution. Ans- 129 : (d) Shows decreased perfusion in McLeod's Syndrome.


Ref: Read the text below
Sol:
In radionucleotide lung scans, macroaggregated human serum albumin is injected intravenously, and becomes
trapped in the pulmonary capillary bed.
The distribution of radioactivity is proportional to pulmonary capillary blood flow. It will, therefore, be
abnormal in pulmonary embolism, and congenital cardiovascular and pulmonary defects.
Acute changes in distribution or perfusion may reflect alterations in pulmonary ventilation. Using inhaled
radioactive xenon (133Xe), the distribution of pulmonary ventilation may be estimated.
After intravenous injection of the xenon dissolved in saline, both pulmonary perfusion and ventilation can be
evaluated by continuous recording of the rate of appearance and disappearance of the xenon over the lung.
The appearance of xenon early after injection is a measure of perfusion, whereas the rate of washout during
breathing is a measure of ventilation.
McCleod's Syndrome consists of a congenital lung cyst.

Correct Answer. d

(130). The following conditions can be inherited as an autosomal dominant:

a. Achondroplasia

b. Prader-Willi syndrome

c. B-thalassaemia

d. Downs syndrome

Solution. Ans- 130 : (a) Achondroplasia


Ref: Read the text below
Sol:
Neurofibromatosis and achondroplasia are single gene Autosomal dominant disorders. B thalassaemia is
recessively inherited.
One copy of the abnormal gene is termed thalassaemia minorand if there are 2 copies of the abnormal gene
the condition which develops is thalassaemia major.
Prader Willi syndrome is a chromosomal disorder characterised by insatiable appetite, hyperglycaemia and
short stature.
Downs syndrome is a chromosomal disorder.

Correct Answer. a

(131). The treatment of choice for the polymyositis is

a. An anticholinesterase drug

b. An immunosuppressive drug

c. Cortisone

d. Thyroidectomy

Solution. Ans 131: (c) Cortisone


Reference: Read the text below
Sol:
Cortisone is the drug of choice for the treatment of polymyositis. If this treatment fails, immunosuppressive drugs can be tried.
The end point of initial therapy is restoration of strength and normalization of muscle enzymes.
Although drug therapy may sometimes be discontinued, most patients require long-term treatment.

Correct Answer. c

Copyright 2014 Delhi Academy of Medical Sciences, All Rights Reserved. 56/85
(132). Staphylococcus aureus endocarditis in intravenous drug abusers

a. Is less common than Staphylococcus epidermitis endocarditis

b. Preferentially involves the tricuspid valve

c. Should not be treated with valve replacement surgery

d. Is commonly associated with murmurs and peripheral stigmata

Solution. Ans 132: (b) Preferentially involves the tricuspid valve


Reference: Read the text below
Sol:
Over half the cases of endocarditis in intravenous drug abusers are caused by Staphylococcus aureus.
The tricuspid valve is preferentially involved (50% of cases), followed by the aortic (25%) and mitral (20%) valves.
Cardiac murmurs and peripheral stigmata are rare with endocarditis in intravenous drug abusers, especially when the tricuspid valve is
involved. Septic and aseptic pulmonary emboli are common.
If positive blood cultures persist on antibiotic therapy for the treatment of endocarditis, surgical valve replacement is necessary in all
cases.

Correct Answer. b

(133). Diagnostic features of allergic bronchopulmonary aspergillosis include all of the following except

a. Eosinophilia

b. Positive skin test for aspergillus fumigatus

c. Precipitans to aspergillus

d. Peripheral bronchiectasis on chest roentgenogram

Solution. Ans 133: (d) Peripheral bronchiectasis on chest roentgenogram


Reference: Read the text below
Sol:
Bronchopulmonary aspergillosis occurs in individuals with extrinsic asthma whose airways become a culture medium for Aspergillus
fumigatus.
Clinical criteria used to establish a diagnosis of bronchopulmonary aspergillosis include a history of pulmonary infiltrates, the presence
of asthmatic symptoms,and a positive immediate skin test to A.fumigatus.
Precipitins to Aspergillus, sputum cultures growing Aspergillus, and eosinophilia are important laboratory findings. Chest
roentgenograms typically show central bronchiectasis.

Correct Answer. d

(134). The factor that signifies a good prognosis in rheumatoid arthritis is

a. Eosinophilia

b. Female sex

c. Thrombocytosis

d. Low rheumatoid factor titers

Solution. Ans 134: (d) Low rheumatoid factor titers


Reference: Read the text below
Sol:
Rheumatoid arthritis tends to be less severe in males than in females.
Also, the disease generally is less severe if associated with an acute onset rather than an insidious onset,which is more common.
Eosinophilia and thrombocytosis are associated with a poorer prognosis.

Correct Answer. d

Copyright 2014 Delhi Academy of Medical Sciences, All Rights Reserved. 57/85
(135). A routine lipid screen in a 32-year-old woman reveals hypertriglyceridemia. Secondary causes to be ruled out exclude

a. Hyperthyroidism

b. Loop diuretics

c. Inhaled beta agonists

d. Excessive alcohol consumption

Solution. Ans 135: (d) Excessive alcohol consumption


Reference: Read the text below
Sol:
The many causes of secondary hypertriglyceridemia include hypothyroidism, chronic renal failure, nephrotic syndrome, diabetes
mellitus, glycogen storage disease, acute stress states, and rarely the dysgammaglobulinemias (multiple myeloma, macroglobulinemia,
and systemic lupus erythematosus).
Drugs that may raise triglyceride levels are thiazide diuretics, beta blockers, estrogens,corticosteroids, and retinoids.
Women who start with modest triglyceride elevations can have enormous increases in their triglyceride levels and face the risk of attacks
of acute pancreatitis when placed on oral contraceptives.
Corticosteroids often raise triglyceride levels, sometimes in association with the development of glucose intolerance.
Hypertriglyceridemia, at times severe, is a common side effect of the use of retinoids.

Correct Answer. d

(136). Which of the following associations is recognised:

a. Pulmonary plethora and pulmonary atresia with intact ventricular septum.

b. Prominent right heart border and ASD.

c. Pericardial calcification and endocardial fibroelastosis.

d. Cottage loaf' heart and cardiomyopathy

Solution. Ans-136 : (b) Prominent right heart border and ASD


Ref: Read the text below
Sol:
In pulmonary atresia with intact ventricular septum there is decreased pulmonary blood flow.
The prominent right heart border occurs in ASD becasue of right atrial hypertrophy. In endocardial fibroelastosis the interior of the
ventricles appear echogenic, but there is no pericardial calcification.
Pericardial calcification is found in constrictive pericarditis.
A 'cottage loaf' heart is characteristic of TAPVD, and selenium deficiency is the cause of Keshan Disease, a rare nutritional
cardiomyopathy that is frequently fatal in the Keshan Province of China.

Correct Answer. b

(137). Congenital hypothyroidism:

a. Cannot be diagnosed clinically before 3 months of age

b. May be associated with an ectopic thyroid gland

c. Needs short term Thyroxine replacement

d. Can present in the newborn period with haemolytic jaundice

Solution. Ans- 137 : (b) May be associated with an ectopic thyroid gland
Ref: Read the text below
Sol:
Patients with congenital hypothyroidism may exhibit non specific clinical features e.g. jaundice and poor feeding however macroglossia
is a fairly specific clinical finding aiding diagnosis.
The commonest causes of congenital hypothyroidism are an absent or ectopic gland. The need for thyroxine is life long.
The hyperbilirubinaemia is conjugated and haemolytic jaundice would not occur.
Short stature and delayed bone age occur.

Correct Answer. b

Copyright 2014 Delhi Academy of Medical Sciences, All Rights Reserved. 58/85
(138). A 50 years old man is admitted to the hospital with acute myocardial infarction. After 12 hours he becomes hypotensive and oliguric. He
is lying comfortably on his back, B.P. is 90/60 mmHg, heart rate is 60 BPM and JVP is 15 cm H2O. The heart sounds are regular without
gallop, murmur or rub and the lungs are clear on auscultation. The next step should be to give:-

a. Intravenous Furosemide

b. Intravenous Fluids

c. Digoxin & Dopamine

d. Norepinephrine and Intraaortic Balloon Pump

Solution. Ans- 138 : (b) Intravenous Fluids


Ref: Read the text below
Sol:
This patient probably has an inferior wall myocardial infarction complicated by right ventricular involvement.
The Cardiac output is probably depressed because of low left heart filling pressure secondary to right ventricular infarction.
The initial treatment should be to administer fluids intravenously.

Correct Answer. b

(139). Which of the following malignancy is associated with congenital hyperkeratosis and pitting of palms and soles?

a. Bladder cancer

b. Gastric cancer

c. Colon cancer

d. Esophageal cancer

Solution. Ans- 139 : (d) Esophageal cancer


Ref: Read the text below
Sol:
Congenital hyperkeratosis and pitting of palms and soles (i.e., tylosis palmaris et plantaris) has been linked with squamous cell
esophageal cancer.

Correct Answer. d

(140). A 58 year old man has recent onset diabetes mellitus, weight loss and a skin rash that is most prominent on the buttocks and that a
dermatologist diagnoses as necrolytic migratory erythema. What is the probable underlying diagnosis?

a. Adrenal adenoma

b. Pituitary GH secreting tumor

c. Glucagonoma

d. Pheochromocytoma

Solution. Ans- 140 : (c) Glucagonoma


Ref: Read the text below
Sol:
Diabetes mellitus, weight loss and necrolytic migratory erythema are virtually diagnostic of a glucagons secreting pancreatic islet cell
tumor (glucagonoma).
Characteristic rash usually starts as an annular erythema at intertriginous & periorificial sites, especially in the groin or buttock

Correct Answer. c

Copyright 2014 Delhi Academy of Medical Sciences, All Rights Reserved. 59/85
(141). All the following are associated with pure red cell aplasia except :-

a. Anterior mediastinal masses

b. Connective tissue disorders

c. Giant pronormoblasts

d. Low erythropoietin levels

Solution. Ans- 141 : (d) Low erythropoietin levels


Ref: Read the text below
Sol:
Pure red cell aplasia is a normochromic, normocytic anemia with absent erythoblasts on the bone marrow, hence the diminished number
or lack of reticulocytes.
The bone marrow shows red cell aplasia and the presence of giant pronormoblasts.
Several conditions have been associated with pure red cell aplasia, including viral infections such as B19 parvovirus (which can have
cytopathic bone marrow changes), HIV, EBV, HTLV, and hepatitis B virus ; malignancies such as thymomas and lymphoma (which often
present with an anterior mediastinal mass) ; connective tissue disorders such as SLE and rheumatoid arthritis (RA) ; pregnancy ; drugs ;
and hereditary disorders.
Erythropoietin levels are elevated because of the anemia.

Correct Answer. d

(142). Haemolytic anemias are characterised by all except :-

a. Reticulocytosis

b. Increase haptoglobulin

c. Hypercellular marrow

d. Increase methemalbuminemia

Solution. Ans- 142 : (b) Increase haptoglobulin


Ref: Read the text below
Sol:

Correct Answer. b

Copyright 2014 Delhi Academy of Medical Sciences, All Rights Reserved. 60/85
(143). Which one of the following is the cause of non cardiogenic pulmonary oedema seen in immunologic blood transfusion reaction :-

a. Antibody to IgA in donor plasma

b. Antibody to donor leukocyte antigen

c. Donor antibody to leukocyte of patient

d. RBC incompatibility

Solution. Ans- 143 : (b) Antibody to donor leukocyte antigen


Ref: Read the text below
Sol:
Transfusion-related acute lung injury
It results from the transfusion of donor plasma that contains high-titer anti-HLA antibodies that bind recipient
leukocytes.
The leukocytes aggregate in the pulmonary vasculature and release mediators that increase capillary
permeability.
The recipient develops symptoms of respiratory compromise and signs of noncardiogenic pulmonary edema,
including bilateral interstitial infiltrates on chest x-ray.
Treatment is supportive, and patients usually recover without sequelae.

Correct Answer. b

(144). The following are characteristics of central fever except that there is :

a. No diurnal variation

b. No sweating

c. Decreased response to external cooling

d. Resistant to antipyretics

Solution. Ans-144: (c) Decreased response to external cooling


Ref.: Harrison -1716
Sol :
The following are characteristics of central fever except that there is decreased response to external cooling.
Central fever : when fever is attributable to a brain lesion that has disturbed hypothalamic temperatureregulating
centre.

Correct Answer. c

(145). Reduced serum iron and iron binding capacity is seen in :

a. Thalassemia

b. Iron deficiency anemia

c. Chronic infections

d. Sideroblastic anemia

Solution. Ans-145: (c) Chronic infections


Ref.: Harrison-633
Sol :
Reduced serum iron and iron binding capacity is seen in chronic infections.

Correct Answer. c

Copyright 2014 Delhi Academy of Medical Sciences, All Rights Reserved. 61/85
(146). Platelet can be stored at :

a. 20-24C for 5 days

b. 20-24C for 8 days

c. 4-8C for 5 days

d. 4-8C for 8 days

Solution. Ans-146: (a) 20-24C for 5 days


Ref.: Harrison 17th/709
Sol :
Platelet can be stored at 20-24C for 5 days

Correct Answer. a

(147). Which of the following statements is true:

a. Brucellosis is characterised by neutrophil leucocytosis.

b. Toxoplasmosis causes visceral larva migrans.

c. Toxoplasmosis causes posterior uveitis.

d. Serological evidence of toxoplasmosis is rare in adults.

Solution. Ans 147: (c) Toxoplasmosis causes posterior uveitis.


Reference: Read the text below
Sol:
Brucellosis is a zoonosis, spreading from infected animals particularly cattle.
There are 4 species, melitensis, abortus, suis, and canis.
Brucella are gram negative bacilli which are fastidious. There is usually a history of exposure, and the symptoms are rather nonspecific
with fever, malaise, arthralgia and depression. 35% have hepatosplenomegaly.
Leukopaenia is common, and 75% have a positive blood culture (90% of bone marrow cultures will be positive). Toxoplasma is most
frequent in farming communities where contact occurs with cats, and patients eat raw meat.
Clinical manifestations include: focal choroidoretinitis or posterior uveitis, optic atrophy, retinal detachment, cataract and glaucoma.

Correct Answer. c

Copyright 2014 Delhi Academy of Medical Sciences, All Rights Reserved. 62/85
(148). A 9 year old boy shows signs of early development of masculine characteristics. This is most likely to be secondary to an abnormal level
of which hormone?

a. Growth hormone

b. Thyroid hormone

c. Elevated 17-oh progesterone

d. Parathyroid hormone

Solution. Ans 148: (c) Elevated 17-oh progesterone


Reference: Read the text below
Sol:
The disease here is adrenogenital syndrome.
Signs and Symptoms:
In females:ambiguous genitalia early appearance of pubic and axillary hair hair, excessive on females deep voice menstrual periods,
abnormal failure to menstruate .
In males: early development of masculine characteristic enlarged penis small testes early appearance of pubic and axillary hair in both
height as children will be taller, but ultimate adult height will be significantly shorter Signs and tests: elevated urinary 17-ketosteroids
normal or decreased urinary 17-hydroxycorticosteroids elevated 17-OH progesterone elevated serum DHEA sulfate X-ray for bone age
(demonstrates markedly advance bone age above chronological age) serum electrolytes (abnormal in infants with salt-losing form of
adrenogenital syndrome)

Correct Answer. c

(149). Which of the following is characteristic of acute hepatitis B:

a. Most patients present with splenomegaly.

b. It confers immunity to hepatitis A.

c. It commonly presents with distal joint arthritis.

d. There is increased infectivity in the presence of the E antigen.

Solution. Ans 149: (d) There is increased infectivity in the presence of the E antigen
Reference: Read the text below
Sol:
Clinical features of hepatitis B are as follows:
Most are asymptomatic.
Symptoms: Lethargy, anorexia, arthralgia, rash (any type), popular acrodermatitis (Gianotti Crosti), polyarthritis, glomerulonephritis,
aplastic anaemia. 25 % have jaundice.
Complications: Acute fulminent hepatitis. Chronic hepatitis. Membranous glomerulonephritis. Hepatitis E antigen is present in the
acute phase and indicates a highly infectious state. Pruritis is characteristic of chronic hepatitis.

Correct Answer. d

(150). Concerning CNS involvement in AIDS:

a. Toxoplasma may give rise to a focal lesion with neurological weakness.

b. The HIV virus can be isolated from the brain of an encephalopathic patient.

c. A diagnostic elevation in the CSF IgM occurs in toxoplasmic infection.

d. Occular involvement may cause blindness.

Solution. Ans 150: (c) A diagnostic elevation in the CSF IgM occurs in toxoplasmic infection
Reference: Read the text below
Sol:
Cerebral toxoplasmosis presents very variably, from an acute encephalopathy to subtle neurological syndromes. It should be considered
in all undiagnosed neurological disease in the under ones, especially if there are retinal lesions.
Characteristic are hydrocephalus, seizures with focal defects, spinal or bulbar palsies, microcephaly, and decreased IQ. Investigations
such as skull x-ray or CT scan show calcification of the periventricular area, tachyzoites in the CSF and positive blood titres.
Pyrimethamine and Sulphadiazine have a synergistic effect in treating it, and folinic acid may be necessary to prevent seizures.

Correct Answer. c

Copyright 2014 Delhi Academy of Medical Sciences, All Rights Reserved. 63/85
(151). Which one of the following is the most common site of Berry aneurysm ?

a. Vertebro-basilar artery

b. Origin of posterior communicating artery

c. Anterior communicating artery

d. Anterior chroidal artery

Solution. Ans-151: (c) Anterior communicating artery


Ref.: Harrison 17th/2532
Sol :
Anterior communicating artery is the most common site of Berry aneurysm

Correct Answer. c

(152). Warm-antibody immunohemolytic anemia is seen in all except :

a. SLE

b. Chronic Lymphocytic Leukemia

c. Rheumatoid Arthritis

d. Infectious mononucleosis

Solution. Ans-152: (d) Infectious mononucleosis


Ref.: Harrison 17th/660
Sol :
Warm-antibody immunohemolytic anemia may be:
Idiopathic, that is, without any known cause
Secondary to another disease, such as an antecedent upper respiratory tract infection, systemic lupus erythematosus or rheumatoid
arthritis or a malignancy, such as chronic lymphocytic leukemia (CLL)
Associated with receiving a drug

Correct Answer. d

(153). A 22-year old man presents with history of bleeding from gums for the last 6 months. On investigation the Hb was found to be 8.2 gm%
TLC 4400/mm, DLC N 64%, L 27%, E 3%, M 6% and platelet count of 20,000/cumm. Which one of the following investigation would be
most useful in establishing the diagnosis :

a. Bleeding time

b. Prothrombin time

c. Partial thromboplastin time

d. Bone marrow examination

Solution. Ans-153: (d) Bone marrow examination


Ref.: Harrison 17th/666
Sol :
In the given question, patients has
o Bleeding from gums & platelet count of 20,000 (thrombocytopenia)
o Hemoglobin 8.2 gm% (Anemia)
o TLC 4400/mm (Leukocytopenia)
Bone marrow examination will be the most useful investigation to look for the cause of pancytopenia

Correct Answer. d

Copyright 2014 Delhi Academy of Medical Sciences, All Rights Reserved. 64/85
(154). The single most powerful predictor of survival in multiple myeloma is :

a. M component production

b. Bone marrow plasmocytosis

c. Serum beta 2-microglobulin level

d. Serum calcium level

Solution. Ans-154: (c) Serum beta 2-microglobulin level


Ref.: Harrison 17th/702-704
Sol :
Serum 2 microglobulin level is single most powerful predictor of survival and can substitute for staging.

Correct Answer. c

(155). Cryoprecipitate is rich in factor :

a. II

b. V

c. VIII

d. Fibrinogen

Solution. Ans-155: (c) VIII


Ref.: Harrison 17th/710
Sol :
Cryoprecipitate is rich in factor VIII.

Correct Answer. c

Copyright 2014 Delhi Academy of Medical Sciences, All Rights Reserved. 65/85
(156). An Indian adult who has never travelled abroad comes with a history of high fever, headache, jaundice, marked oliguria and shock
withTLC of 16,000/cumm. The most likely diagnosis is :

a. Viral hepatitis

b. Leptospirosis (Wells disease)

c. Yellow fever

d. Hemolytic uraemic syndrome

Solution. Ans-156: (b) Leptospirosis (Wells disease)


Ref.: Harrison 17th/1050
Sol :
Leptospirosis is caused by exposure to several types of the Leptospira bacteria, which can be found in fresh water that has been
contaminated by animal urine. It occurs in warmer climates.
It is not spread from person to person, except in vary rare cases when it is transmitted through breast milk or from a mother to her
unborn child.
Risk factors include:
Occupational exposure -- farmers, ranchers, slaughterhouse workers, trappers, veterinarians, loggers, sewer workers, rice field workers,
and military personnel
Recreational activities -- fresh water swimming, canoeing, kayaking, and trail biking in warm areas
Household exposure -- pet dogs, domesticated livestock, rainwater catchment systems, and infected rodents
Leptospirosis
Symptoms
Symptoms can take 2 - 26 days (average 10 days) to develop, and may include:
Dry cough
Fever
Headache
Muscle pain
Nausea, vomiting, and diarrhea
Shaking chills
Less common symptoms include:
Abdominal pain
Abnormal lung sounds
Bone pain
Conjunctivitis
Enlarged lymph glands
Enlarged spleen or liver
Joint aches
Muscle rigidity
Muscle tenderness
Skin rash
Sore throat

Correct Answer. b

Copyright 2014 Delhi Academy of Medical Sciences, All Rights Reserved. 66/85
(157). The diagnostic characteristics of Plasmodium falciparum (see figure) are best described by which one of the following statements?

a. A period of 72 h is required for the development of the mature schizont, which resembles a rosette with only 8 to 10 oval merozoites

b. An important diagnostic feature is the irregular appearance of the edges of the infected red blood cell

c. The signet-ringshaped trophozoite is irregular in shape with ameboid extensions of the cytoplasm

d. Except in infections with very high parasitemia, only ring forms of early trophozoites and the gametocytes are seen in the peripheral
bloo

Solution. Ans-157: (d) Except in infections with very high parasitemia, only ring forms of early trophozoites and the gametocytes are
seen in the peripheral blood
Ref: Read the text below
Sol:
Plasmodium falciparum infection is distinguished by the appearance of ring forms of early trophozoites and gametocytes, both of which
can be found in the peripheral blood. The size of the RBC is usually normal.
Double dots in the rings are common

Correct Answer. d

(158). Which of the following is the earliest manifestation of Cushing syndrome?

a. Loss of diurnal variation

b. Increased ACTH

c. Increased plasma cortisol

d. Increased urinary metabolites of cortisol

Solution. Ans- 158 : (a) Loss of diurnal variation


Ref: Read the text below
Sol:
Owing to circadian variability, plasma cortisol and to a certain extent ACTH determinations are not meaningful when determined in
isolation, but the absence of normal fall of plasma cortisol at midnight (loss of diurnal variation) is consistent with Cushing's syndrome

Correct Answer. a

Copyright 2014 Delhi Academy of Medical Sciences, All Rights Reserved. 67/85
(159). The first organ to become predominantly lymphoid is:

a. Thymus

b. Lymph node

c. Spleen

d. MALT

Solution. Ans- 159 : (a) Thymus


Ref: Read the text below
Sol:
By the eight week, mesenchymal stem cells from the yolk sac, fetal liver & bone marrow reach the thymus & differentiate into the
thymic lymphoid cells (thymocytes).
It is thus the first organ in all animal species to become predominantly lymphoid.
The spleen is the largest of the lymphoid organs

Correct Answer. a

(160). Which of the following causes malabsorption diarrhea due to infestation?

a. Ascaris lumbricoides

b. Giardia lamblia

c. Entamoeba histolytica

d. Hookworms

Solution. Ans- 160 : (b) Giardia lamblia


Ref: Read the text below
Sol:
Giardia lamblia is a protozoal parasite
Inhabits small intestine of humans & other mammals
Can cause asymptomatic carriage to fulminant diarrhea & malabsorption
Cause loss of brush border enzyme activities.

Correct Answer. b

(161). Which of the following infection does not involve nervous system?

a. Taenia solium

b. Acanthamoeba

c. Naegleria

d. Trichinella spiralis

Solution. Ans- 161 : (d) Trichinella spiralis


Ref: Read the text below
Sol:
Taenia solium-Neurocysticercosis
Acanthamoeba-Primary amoebic meningoencephalitis
Trichinella spiralis is not a neuroparasite and is not found in the brain at any time during its life cycle.
It does not produce any neurological manifestations

Correct Answer. d

Copyright 2014 Delhi Academy of Medical Sciences, All Rights Reserved. 68/85
(162). A 21 year old married women Sharmila presented with burning of micturation since 3 days. On investigations gram positive cocci were
found in urine which were coagulase negative and resistant to nalidixic acid & novobiocin. Most likely causative organism is:

a. Staphylococcus epidermidis

b. Staphylococcus saprophyticus

c. Micrococci

d. Streptococcus agalactiae

Solution. Ans- 162 : (a) Staphylococcus saprophyticus


Ref: Read the text below
Sol:
Staphylococcus saprophyticus is a common etiologic agent for urinary tract infection in sexually active young women.
It is gram positive & coagulase negative.
The infecting strains are usually sensitive to most common antibiotics, except nalidixic acid & novobiocin.

Correct Answer. a

(163). Pneumocystis carinii:

a. Can cause pneumonia with very few signs on chest x-ray.

b. Is an obligate intracellular organism.

c. May cause extrapulmonary infection.

d. Is usually diagnosed by finding a rising titre of neutralising antibodies.

Solution. Ans- 163 : (c) May cause extrapulmonary infection


Ref: Read the text below
Sol:
Pneumocystis carinii pneumoniae is a life-threatening infection occurring in immunosuppressed hosts. With rare exceptions, the
organism is localised to the lungs. It is a common extracellular parasite, that has attributes of both fungi and protozoa. It affects 40% of
infants and children with AIDS and 12% of those with leukaemia if not
prophylaxis is given. Cell mediated immunity is the major defence mechanism, so PCP pneumonia is common in
those with severe combined immune deficiency, while it is rarely found in X-linked agammaglobulinaemia.
There are 2 clinical forms:
Epidermic infantile: 3-6 month old infants, with subtle onset of tachypnoea but no fever.
Progressive recession and increased work of breathing with cyanosis.
Sporadic: Children and adults with immune deficiency. Abrupt fever, tachypnoea, dyspnoea and cough with
cyanosis.
Added sounds are usually minimal in both forms. The chest x-ray reveals bilateral diffuse alveolitis, initially
perihilar, progressing peripherally, and tended to spare the apical areas. Definitive diagnosis requires
demonstration of the organism in the lung by bronchio-alveolar lavage, tracheal aspirates, or bronchial brushings,
transbronchial lung biopsy, needle aspiration or open lung biopsy. Treatment is high dose septrin plus steroids.

Correct Answer. c

Copyright 2014 Delhi Academy of Medical Sciences, All Rights Reserved. 69/85
(164). In patients with hypertrophic cardiomyopathy maximum mutations are found in which gene:

a. b - myosin heavy chain

b. Elastin

c. a tropomyosin

d. Troponin T

Solution. Ans- 164 : (a) b - myosin heavy chain


Ref: Read the text below
Sol:
Mutations in gene for b - Mysoin heavy chain are associated with 40% of the families with hypertrophic
cardiomyopathy.
Troponin T mutations - 15% of the families
- tropomyosin mutations ~5% of the families

Correct Answer. a

(165). All of the following enhances insulin sensitivity; except :-

a. Adiponectin (or ACRP 30)

b. Leptin

c. TNF-

d. Thiazolidinediones

Solution. Ans- 165 : (c) TNF-


Ref: Read the text below
Sol:

Correct Answer. c

(166). Pulsus paradoxus is found in all of following except:

a. A severe asthmatic attack

b. Severe left ventricular failure

c. Myocardial disease

d. Constrictive pericarditis

Solution. Ans- 166 : (b) Severe left ventricular failure


Ref: Read the text below
Sol:
Pulsus paradoxus is the exaggerated fall of pressure associated with inspiration due to disease process like
asthma, constrictive pericarditis, tamponade and left ventricular disease (cardiomyopathy).
It is not a feature of left ventricular failure

Correct Answer. b

Copyright 2014 Delhi Academy of Medical Sciences, All Rights Reserved. 70/85
(167). Regarding inhaler devices:

a. Metered dose inhalers can usually be used from the age of about 7 years.

b. The Spinhaler requires co-ordination of device actuation with inhalation.

c. Salbutamol can be used with the Nebuhaler.

d. A face mask can be attached to a spacer, so that it can be used in infants

Solution. Ans- 167 : (d) A face mask can be attached to a spacer, so that it can be used in infants.
Ref: Read the text below
Sol:
Metered dose inhalers require great co-ordination, and start to be used effectively in children only from about
the age of 12 or 13.
Most other devices are specific to individual manufacturers, and the drug chosen is more on the basis of the
patient's ability to use the inhalation device than on the characteristics of the drug itself.
This is because in a chronic disease such as asthma, compliance and ability to use the drug dispensing device
makes by far the greatest difference in achieving adequate control of symptoms.
A number of inhaler devices have been developed which do not require co-ordination of actuation with
inhalation. These include:
Turbohaler: Terbutaline, Budesonide. From 3 years.
Spinhaler: Cromoglycate. From 3 years.
Diskhaler: Salbutamol, Beclomethasone. From 3 years.
Spacer devices include:
Nebuhaler: Terbutaline, Budesonide. From infancy (with mask).
Volumatic: Salbutamol, Beclomethasone. From infancy (with mask).
Nebulisers can be used at all ages, but recent research suggest they are no more effective than a spacer device with a similar dose of
drug given, even in acute asthma.

Correct Answer. d

(168). Which of the following is characteristic feature of Marfan's syndrome:

a. Autosomal recessive inheritance

b. Pulmonary stenosis

c. Ectopia Lentis

d. Short stature

Solution. Ans- 168 : (c) Ectopia Lentis


Ref: Read the text below
Sol:
Marfan's Syndrome is autosomal dominant.
The characteristic features are tall stature, mainly due to increase in limb length, arachnodactyly, positive Steinburg's sign (thumb
extends beyond the folded fist), hyperextensible joints, high arched palate, lens dislocation, severe myopia.
Chest deformities and scoliosis are common. The major complication is cardiovascular because of medial degeneration. This results in
aortic dissection, aortic and mitral valve prolapses with regurgitation. Echocardiographic monitoring is recommended.
The subiscule height measures the sitting height, and the ratio between the sitting height and the total height is altered in
disproportionate increased or short stature.

Correct Answer. c

Copyright 2014 Delhi Academy of Medical Sciences, All Rights Reserved. 71/85
(169). Antimitochondrial antibodies are found in 90% of patients with

a. Primary biliary cirrhosis

b. Chronic active hepatitis

c. Seminoma

d. Choriocarcinoma

Solution. Ans 169: (a) Primary biliary cirrhosis


Reference: Read the text below
Sol:
The most characteristic immunologic abnormality associated with primary biliary cirrhosis is the presence of antimitochondrial
antibodies, which are found in 90% of affected individuals.
This antibody is predominantly IgG. Antimitochondrial antibodies also can be detected in persons with other liver diseases but with a
much lower frequency (less than 10%) and usually in low titer.

Correct Answer. a

(170). Common findings in hemochromatosis include all of the following except

a. Diabetes mellitus

b. Diabetes insipidus

c. Impotence

d. Hepatocellular carcinoma

Solution. Ans 170: (b) Diabetes insipidus


Reference: Read the text below
Sol:
In hemochromatosis the principal organs involved with excessive iron storage,leading to parenchymal damage and fibrosis,are the liver,
pancreas, heart, and gonads.
Diabetes occurs in the majority of patients and more than 80% demonstrate abnormalities of glucose tolerance. Testicular atrophy and
fibrosis cause impotence.
Diabetes insipidus does not occur because the pituitary and hypothalamus are not involved. Cirrhosis is usually present when the
diagnosis is established,and hepatocellular carcinoma develops in 25 to 40% of patients.
Chondrocalcinosis (pseudogout) is found in 25 to 50% of patients. Other common manifestations include cardiac involvement with
congestive heart failure and arrhythmias.
Melanin deposition causes increased skin pigmentation with a characteristic slate gray color

Correct Answer. b

(171). Mitral valve prolapse can be described by which of the following statements?

a. Most affected individuals are asymptomatic

b. Men are affected more often than women

c. Ventricular arrhythmias are common

d. All of the above

Solution. Ans 171: (a) Most affected individuals are asymptomatic


Reference: Read the text below
Sol:
Most patients with mitral valve prolapsed are asymptomatic.
Ventricular arrhythmias are uncommon, and mitral valve regurgitation and congestive heart failure develop infrequently.
Women are affected more commonly than men.

Correct Answer. a

Copyright 2014 Delhi Academy of Medical Sciences, All Rights Reserved. 72/85
(172). All of the following statement regarding Diabetes mellitus in children are true except:

a. Can often be managed with oral hypoglycaemic agents

b. May be associated with prior mumps infection

c. Often remits for a variable period of time

d. Has a peak incidence at 10-12 years of age

Solution. Ans- 172 : (a) Can often be managed with oral hypoglycaemic agents
Ref: Read the text below
Sol:
Diabetes mellitus commonly presents with ketoacidosis in children, but not always.
Though the incidence of type 2 diabetes in children is increasing, the most common cause is type 1 diabetes and insulin is required.
The onset of type 1 Diabetes sometimes coincides with or follows a viral infection such as mumps, measles, rubella, CMV or influenza.
The initial destruction of beta cells may temporarily halt and there may be a phase of islet cell regeneration which results in a
honeymoon period of remission.
The peak incidence is at aged 10-12 years and most present around autumn and winter.

Correct Answer. a

(173). Where are the most common drivers of atrial fibrillation anatomically located?

a. Left atrial appendage

b. Mitral annulus

c. Pulmonary vein orifice

d. Sinus venosus

Solution. Ans- 173 : (c) Pulmonary vein orifice


Ref: Read the text below
Sol:
The mechanisms for atrial fibrillation initiation and maintenance are still debated; however, there are anatomic structures that play a
role in both of these processes.
Muscularized tissue at the orifices of the pulmonary vein inlets are the predominant anatomic drivers of atrial fibrillation, although
metabolic disturbances (e.g., hyperthyroidism, inflammation, infection) are also very common.
Radiofrequency ablation of the tissue in the area of the pulmonary vein inlets can terminate atrial fibrillation; however, recurrences are
not uncommon and other anatomic drivers may be present.
The left atrial appendage is an important site of thrombus formation in patients with atrial fibrillation. Any focus within the left or right
atrium can be a focus of reentry of focal atrial tachycardia, including the mitral annulus or sinus venosus.
Increased automaticity of the sinus node is the mechanism for sinus tachycardia.

Correct Answer. c

Copyright 2014 Delhi Academy of Medical Sciences, All Rights Reserved. 73/85
(174). When deciding whether to initiate anticoagulation for a patient with atrial fibrillation, which of the following factors is least important?

a. Age

b. History of diabetes

c. Mitral stenosis

d. Use of antiarrhythmic medications

Solution. Ans- 174 : (d) Use of antiarrhythmic medications


Ref: Read the text below
Sol:
A common risk scoring system for anticoagulant use in patients with atrial fibrillation is the CHADS2 scoring system; C = congestive
heart failure, H = hypertension, A = age >75 years, D = diabetes, S = history of stroke.
The presence of any of these risk factors assigns a score of 1, except for stroke, which is worth 2 points.
Low-risk patients (score of 0, stroke risk 0.5%/year without warfarin) can be managed with aspirin alone.
High-risk patients (score 3, stroke risk 5.2%/year without warfarin) should be managed with warfarin.
Intermediate-risk patients (score 1 or 2, stroke risk 1.5% 2.5%/year without warfarin) may be managed with aspirin or warfarin,
depending upon the clinician's assessment of risk, the ability to monitor the intensity of anticoagulation, the patient's risk of bleeding
with an

Correct Answer. d

(175). Which of the following electrocardiographic findings suggests a focal atrial tachycardia as opposed to an automatic atrial tachycardia
(e.g., sinus tachycardia)?

a. Initiation of tachycardia with programmed stimulation

b. One P-wave morphology

c. Slow-onset and termination phase

d. Slowing of the rate with adenosine infusion

Solution. Ans- 175 : (a) Initiation of tachycardia with programmed stimulation


Ref: Read the text below
Sol:
Focal atrial tachycardias can be divided into two categories based on mechanism: automatic and reentry.
Sinus tachycardia is the classic automatic tachycardia in which onset and termination have a warm-up and slow-down period,
respectively.
The P-wave morphology, which initiates the tachycardia of automatic tachycardias, is the same as the P wave of the tachycardia,
whereas the initiating P wave of focal reentrant atrial tachycardia is usually different from those of the tachycardia.
Automatic tachycardias are not reliably initiated by programmed stimulation during an electrophysiologic study, whereas reentrant
atrial tachycardias can be initiated by programmed stimulation or premature beats.
Adenosine receptors within sinus and atrioventricular nodal tissue are thought to account for the ability of this drug to slow and
terminate arrhythmias involving these structures

Correct Answer. a

Copyright 2014 Delhi Academy of Medical Sciences, All Rights Reserved. 74/85
(176). 45 yrs old male presents with wide-complex tachycardia. The patient has a history of Wolff- Parkinson- White (WPW) syndrome. Which
medication is the most effective for treating this patients tachycardia?

a. Adenosine

b. Digoxin

c. Diltiazem

d. Procainamide

Solution. Ans- 176 : (d) Procainamide


Ref: Read the text below
Sol:
Tachycardias that involve an accessory pathway, like WPW, are at risk for degeneration into 1:1 atrial:ventricular conduction down the
accessory pathway and subsequent ventricular tachycardia or fibrillation.
If the circuit can conduct anterogradely (i.e., down the accessory pathway, up the His-Purkinje tract), then atrioventricular (AV) nodal
blocking agents can precipitate ventricular tachycardia.
With a wide-complex tachycardia, the AV nodal blocking agents (adenosine, digoxin, diltiazem, verapamil) will not be as effective as the
class 1a antiarrhythmic agent procaine mide.
Lidocaine and amiodarone would also be effective agents for treating stable widecomplex tachycardias.

Correct Answer. d

(177). All of the following findings on echocardiographic assessment of patients with congestive heart failure with preserved ejection fraction
are relevant except

a. Atrial fibrillation

b. Left atrial dilatation

c. Left ventricular wall thickness

d. Systolic anterior motion of the mitral valve

Solution. Ans- 177 : (d) Systolic anterior motion of the mitral valve
Ref: Read the text below
Sol:
Heart failure with a preserved ejection fraction is very common but can be challenging to evaluate serially.
Each of the described parameters gives important adjunct information regarding heart function in this type of patient.
Left atrial dilatation often implies a chronic elevation in left ventricular diastolic pressures as the atria is relatively compliant and will
dilate in this setting.
Atrial fibrillation is easily seen on echocardiography and is problematic in these patients as they are often dependent on their atrial kick
to maintain preload and therefore cardiac output.
Left ventricular wall thickness and diastolic filling may imply severity and duration of disease.
Systolic anterior motion of the mitral valve with asymmetric septal hypertrophy is a characteristic echocardiographic finding in
hypertrophic cardiomyopathy.

Correct Answer. d

Copyright 2014 Delhi Academy of Medical Sciences, All Rights Reserved. 75/85
(178). All of the following medications have been shown to worsen heart failure in patients with left ventricular systolic dysfunction except

a. Angiotensin receptor blockers

b. Calcium channel antagonists

c. Nonsteroidal anti-inflammatory drugs (NSAIDs)

d. Sotalol

Solution. Ans- 178 : (a) Angiotensin receptor blockers


Ref: Read the text below
Sol:
Angiotensin receptor blockers (ARBs) are useful in heart failure patients who do not tolerate angiotensin-converting enzyme inhibitors
due to cough or other side effects.
Inhibition of the renin-angiotensin pathway reduces left ventricular afterload and remodeling.
They have been shown to improve symptoms and exercise capacity and to reduce need for hospitalization and mortality in patients with
systolic heart failure. Calcium channel blockers, particularly first-generation medications, may worsen function in patients with systolic
dysfunction.
Thiazolidinediones (rosiglitazone, pioglitazone) are associated with fluid retention and may worsen heart failure.
NSAID use in patients with a reduced cardiac output may cause acute renal failure. Sotalol has been shown to increase mortality in
patients with left ventricular dysfunction

Correct Answer. a

(179). A 65-year-old male with mild congestive heart failure is to receive total hip replacement. He has no other underlying diseases and no
history of hypertension, recent surgery, or bleeding disorder. The best approach to prevention of pulmonary embolus in this patient is

a. Aspirin 75 mg/d

b. Aspirin 325 mg/d

c. Warfarin with INR of 2 to 3

d. Early ambulation

Solution. Ans- 179 : (c) Warfarin with INR of 2 to 3


Ref: Read the text below
Sol:
Warfarin is the principal agent recommended for the prophylaxis of acute pulmonary embolus in patients who receive total hip
replacement.
Warfarin is started preoperatively, and the daily dose is adjusted to maintain an international normalized ratio (INR) of 2 to 2.5. Low-
molecular-weight heparin given twice daily subcutaneously is also a recommended regimen.
The value of aspirin in this setting is unclear.
Early ambulation and elastic stockings are also important in preventing thromboembolism, but are not adequate in themselves in this
high-risk situation.

Correct Answer. c

(180). Meningococcal infection occur due to deficiency of which of the following complement component:-

a. C5 C8

b. C9

c. C1, C2, C4

d. C1 inhibitor

Solution. Ans- 180 : (a) C5 C8


Ref: Read the text below
Sol:
Bacteremia, mainly with gram negative diplococci, Neisseria meningitidis & N.gonorrhoeae & toxoplasmosis is caused due to deficiency
of complement component C5C8
Deficiency of early complement component leads to SLE & collagen vascular diseases.
Deficiency of C1 inhibitor leads to angioneurotic edema.

Correct Answer. a

Copyright 2014 Delhi Academy of Medical Sciences, All Rights Reserved. 76/85
(181). Ultraviolet light causes mutation by which of the following mechanism :-

a. By causing frameshift mutation

b. By linking neighbouring thymine bases to form dimer

c. By altering structure of bases in DNA

d. By deleting a single base pair

Solution. Ans- 181 : (b) By linking neighbouring thymine bases to form dimer
Ref: Read the text below
Sol:
The frequency of mutation is greatly enhanced by exposure of cells to mutagens.
UV light damages DNA by linking neighbouring thymine bases to form dimer.
Chemical mutagens may act by altering either the chemical or physical structure of DNA.
They may also cause frameshift mutation.

Correct Answer. b

(182). The feature of Turners syndrome include:

a. Mental retardation

b. Karyotype XXY

c. Webbing of the neck

d. None

Solution. Ans- 182 : (c) Webbing of the neck


Ref: Read the text below
Sol:
Short stature (with height of >2.5 standard deviations below the mean height value for age) is an important phenotypic feature of the
condition and it is not due to deficiency of growth hormone, thyroid hormone, adrenal or gonadal steroids but may improve following the
administration of growth hormone.
In general patients with gonadal dysgenesis do not differ from siblings in overall intelligence. The karyotype is XO or XO/XX in mosaics.
Webbing of the neck, lymphoedema of the distal extremities, ascites and pleural effusions are features.
Cardiovascular abnormalities other than coarctation include bicuspid aortic valve, mitral valve prolapse and aortic root dilatation.

Correct Answer. c

(183). All of the following are electrocardiographic clues supporting the diagnosis of ventriculartachycardia except

a. Capture beats

b. Concordance of QRS complex in all precordial leads

c. QRS duration during tachycardia shorter than during sinus rhythm

d. RSR' pattern in V1

Solution. Ans- 183 : (d) RSR' pattern in V1


Ref: Read the text below
Sol:
Diagnosing ventricular tachycardia based on the surface electrocardiogram is challenging.
In the presence of any interventricular conduction delay, the sinus rhythm QRS duration may be prolonged while the ventricular
tachycardia depolarization pattern may originate from closer to the interventricular septum, resulting in a narrower QRS complex. Other
clues supporting ventricular tachycardia include a bizarre
QRS pattern that does not mimic typical left bundle branch block (LBBB) or right bundle branch block (RBBB) QRS complexes, delayed
activation of the initial phase of the QRS complex, a frontal plane axis between 90 and 180 and a prolonged QRS duration in the
presence of an LBBB or RBBB.
Fusion beats and atrial capture signify atrioventricular dissociation and are the most specific clues for ventricular tachycardia; however,
they are also the least commonly found.

Correct Answer. d

Copyright 2014 Delhi Academy of Medical Sciences, All Rights Reserved. 77/85
(184). Hypophosphataemia is associated with:

a. Vitamin D therapy

b. Diabetic ketoacidosis

c. Metabolic alkalosis

d. Minimal change glomerulonephritis

Solution. Ans- 184 : (b) Diabetic ketoacidosis


Ref: Read the text below
Sol:
Hypophosphataemia is associated with:
Decreased intake: premature neonate, starvation or protein energy malnutrition, malabsorption.
Fluid or electrolyte shifts: respiratory/metabolic alkalosis, diabetic ketoacidosis, steroid therapy.
Increased losses: primary and tertiary hyperparathyroidism, renal tubular defects, diuretic
therapy, post intravenous fluids.
Combination: Vitamin D deficiency, Vitamin D resistant rickets.

Correct Answer. b

(185). A grossly obese child age 13 years would have an increased likelihood of:

a. Developing a slipped upper femoral epiphysis

b. Being prone to respiratory infections

c. Having been weaned early

d. Elevated fasting blood sugar

Solution. Ans- 185 : (a) Developing a slipped upper femoral epiphysis


Ref: Read the text below
Sol:
Slipped upper femoral epiphyses is associated with obesity and growth hormone administration.
Respiratory infections are not associated. The single greatest risk factor for childhood obesity is maternal obesity.
Weaning early is not a risk factor.
Gross childhood obesity is associated with increased risk of type 2 diabetes, but this would manifest later.

Correct Answer. a

Copyright 2014 Delhi Academy of Medical Sciences, All Rights Reserved. 78/85
(186). Which of the following is the most appropriate therapy for a 60-year-old male with 2 weeks of productive cough, fever, shortness of
breath, and the chest radiogram as shown in the following figure?

a. Cephalexin

b. Ciprofloxacin

c. Clindamycin

d. Penicillin

Solution. Ans-186: (c) Clindamycin


Ref: Read the text below
Sol:
The radiograph describes a lung abscess that most likely is due to anaerobic infection. The anaerobes involved are most likely oral, but
Bacteroides fragilis is isolated in up to 10% of cases.
Vancomycin, ciprofloxacin, and cephalexin have no significant activity against anaerobes.
Most oral anaerobic strains have the capacity to produce -lactamase. For many years penicillin was considered the standard treatment
for anaerobic lung infections.
However, clinical studies have demonstrated the superiority of clindamycin over penicillin in the treatment of lung abscess. When there
are contraindications to clindamycin, penicillin plus metronidazole is likely to be as effective as clindamycin.

Correct Answer. c

(187). 100% oxygen improves cyanosis in all except :

a. Tetralogy of Fallot

b. Bronchial asthma

c. Eosinophilic pneumonia

d. Interstitial lung disease

Solution. Ans-187: (a) Tetralogy of Fallot


Ref.: Harrison 17th/1592
Sol :

Correct Answer. a

Copyright 2014 Delhi Academy of Medical Sciences, All Rights Reserved. 79/85
(188). What is the main mode of action of beta-blockers in controlling hypertension ?

a. Decrease cardiac input

b. Slow the heart rate

c. Increase the cardiac force of contraction

d. Increase cardiac output

Solution. Ans 188: (b) Slow the heart rate


Reference: Read the text below
Sol:
Beta-blockers act by blocking the action of noradrenaline at b adrenoceptors throughout the circulatory system and elsewhere.
Their major effect is to slow the heart rate and reduce its force of contraction. beta-blockers also cause some reduction in renin release
and central sympathetic tone.

Correct Answer. b

(189). In a patient with receptor-positive breast carcinoma what is the role of anastrozole?

a. Blocks the ovarian production of oestrogens

b. It is a nonsteroidal aromatase enhancer

c. Stimulates the production of progesterone

d. Blocks the conversion of androgens to estrogens

Solution. Ans 189: (d) Blocks the conversion of androgens to estrogens


Reference: Read the text below
Sol:
Anastrozole is indicated as adjuvant treatment of oestrogen receptor-positive early breast cancer.
Anastrozole is a reversible (Type II), nonsteroidal aromatase inhibitor. Aromatase catalyzes the final and rate-limiting step in the
conversion of androgens to estrogens in peripheral tissues. This occurs mainly in adipose tissue, but also in normal and malignant breast
tissues, and provides the main source of estrogen in postmenopausal women. The goal of hormone therapy in breast cancer is to deprive
tumour cells of estrogens, which are implicated in the development or progression of tumours. Maximal estrogen suppression is
produced by a 1mg dose.
Estrogen suppression is maintained for up to six days after discontinuing anastrozole. Differences in the mechanism of action may
contribute to the apparent lack of cross-resistance between steroidal (eg, exemestane) and nonsteroidal (eg, anastrozole and letrozole)
aromatase inhibitors. Highly selective blockade
of aromatase does not interfere with the production of other steroids (eg, adrenal corticosteroids, aldosterone) or thyroid stimulating
hormone.
Anastrozole does not have progestogenic, androgenic or estrogenic activity.

Correct Answer. d

Copyright 2014 Delhi Academy of Medical Sciences, All Rights Reserved. 80/85
(190). A critically ill and immuncompromised patient receiving assisted ventilation develops a Pseudomonas aeruginosa pneumonia. The most
appropriate treament would include

a. Intravenous Trimethoprim/Sulfamethoxazole

b. Combination of ticarcillin and gentamicin

c. 1-time dose of metronidazole

d. IM ceftriaxone plus doxycycline

Solution. Ans 190: (b) Combination of ticarcillin and gentamicin


Reference: Read the text below
Sol:
Although community-acquired cases of pneumonia in previously healthy adults have been described, the vast majority of cases occur in
hospitalized patients.
Risk factors for the development of P aeruginosa pneumonia are: (1) mechanical ventilation in an intensive care setting; (2) cancer,
especially with concomitant neutropenia; (3) hypogammaglobulinemia (primary or acquired); (4) cystic fibrosis; and (5) prior
antimicrobial-agent therapy.
Notably, the presence of HIV infection has not been associated with a greater risk for P aeruginosa pneumonia For the bacteremic,
immunosuppressed, or critically ill patient with Pseudomonas pneumonia, combination therapy is recommended.
The current standard for treatment is an antipseudomonal beta-lactam agent combined with an aminoglycoside.

Correct Answer. b

(191). Propylthiouracil is useful in the treatment of hyperthyroidism. Its action does not include

a. Blocking the organification and coupling process

b. Inhibiting iodine trapping and organification

c. Blocking proteolysis of hormone from thyroglobulin

d. Blocking hormone release

Solution. Ans 191: (b) Inhibiting iodine trapping and organification


Reference: Read the text below
Sol:
Steps involved in the synthesis of thyroid hormone include
(1) trapping of iodine by follicular cells;
(2) organification of the iodine by linking it to tyrosines on thyroglobulin;
(3) coupling two of these tyrosines to form iodothyronines; and
(4) proteolysis and release of thyroid hormones from thyroglobulin stored as colloid in the follicular lumen.Tetraiodothyronine (T4) is
converted in peripheral tissues to triiodothyronine (T3).
Propylthiouracil blocks organification and coupling within the thyroid and also reduces the conversion of T4 to T3.

Correct Answer. b

Copyright 2014 Delhi Academy of Medical Sciences, All Rights Reserved. 81/85
(192). A late complication of the acquired immunodeficiency syndrome is

a. Candida esophagitis

b. Pneumocystis carinii pneumonia

c. Cognitive impairment

d. Kaposis sarcoma

Solution. Ans 192: (c) Cognitive impairment


Reference: Read the text below
Sol:
Persons with acquired immunodeficiency syndrome (AIDS) frequently present with opportunistic infections, especially Pneumocystis
carinii pneumonia.
Also common in these persons is the otherwise rare dermal malignancy, Kaposis sarcoma. One of the most striking early clinical
features is progressive weight loss out of proportion to any reduction in food intake.
Candida esophagitis usually occurs at the onset of the disease or in the course of therapy for opportunistic infections.
Cognitive impairment from AIDS-related dementia, however, usually occurs later in the course of the disease.

Correct Answer. c

(193). Bacteremia resulting from contaminated intravenous fluids is most likely to be caused by

a. Staphylococcus aureus

b. Staphylococcus epidermidis

c. Escherichia coli

d. Enterobacter agglomerans

Solution. Ans 193: (d) Enterobacter agglomerans


Reference: Read the text below
Sol:
Infections with contaminated intravenous infusions involve organisms that are potentially pathogenic and survive well in water.
These include Enterobacter agglomerans,Pseudomonas cepacia, Citrobacter freundii, Klebsiella,and Serratia.
A large-scale national epidemic involved Enterobacter agglomerans,which became suspect when a large number of patients receiving IV
fluids from one supplier developed bacteremia with this relatively uncommon blood culture isolate.

Correct Answer. d

(194). A 43-year-old man complains of fatigue,headache, anorexia, and a dull ache in the left upper quadrant of his abdomen. Examination
reveals a mass extending just below the left costal margin; the mass is presumed to be an enlarged spleen. Which of the following would
not be a possible cause of this mans illness?

a. Polycythemia vera

b. Idiopathic thrombocytopenic purpura

c. Folate deficiency

d. Brucellosis

Solution. Ans 194: (c) Folate deficiency


Reference: Read the text below
Sol:
The mass extending below the left costal margin is likely an enlarged spleen.
Splenomegaly has an extensive differential diagnosis that falls into several general categories: infections (eg, acute bacterial
infections),diseases of disordered immunoregulation (eg, Feltys syndrome), conditions with disordered splenic blood flow (eg, cirrhosis
and idiopathic thrombocytopenic purpura), diseases with abnormal erythrocytes (eg, spherocytosis),infiltration by abnormal material (eg,
amyloidosis), extramedullary hematopoiesis (polycythemia vera), and malignant tumors (eg, lymphoma).
Folate deficiency causes anemia but not splenomegaly

Correct Answer. c

Copyright 2014 Delhi Academy of Medical Sciences, All Rights Reserved. 82/85
(195). Recognised features of infectious mononucleosis include all of the following except :

a. Palatal petechial haemorrhages

b. Aseptic meningitis

c. Vesicular rash on neck and trunk

d. Raised asparatate amino transferase

Solution. Ans 195: (c) Vesicular rash on neck and trunk


Reference: Read the text below
Sol:
Incubation period of infectious mononucleosis is 30-60 days. Results of infection:
1. Asymptomatic (most)
2. Simple infection: fever, malaise, headache, myalgia, worsening sore throat abdominal pain. Splenic enlargement in 50% to 2-3cm. 90%
have lymphadenopathy and 10% liver enlargement. Severe pharyngitis with exudates and petechiae is often present, and maculopapular
rash will occur in 80% given Amoxycillin.
3. Complications:
Splenic rupture with trauma (<0.2%).
Stridor or respiratory distress, which may be treated with steroids.
CNS: ataxia, fits, Alice in Wonderland Syndrome, Guillain Barre.
Haematology: Haemolytic anaemia, aplastic anaemia.
Other: Myocarditis, intesitial pneumonitis, pancreatitis, parotitis and orchitis

Correct Answer. c

(196). In ataxia telangiectasia:

a. Inheritance is X-linked recessive.

b. The usual presentation is with lower respiratory infections.

c. Chromosome fragility is increased.

d. The immunodeficiency primarily affects phagocyte function.

Solution. Ans 196: (c) Chromosome fragility is increased


Reference: Read the text below
Sol:
Ataxia telangiectasia is an autosomal recessive disorder, with a defect that has now been localised to chromosome 11q 22-23.
Ataxia occurs when a child first begins to walk, and subsequently, recurrent sino-pulmonary infections occur.
Telangiectasiae become evident at 3-6 years, because of the DNA repair defects (similar defects are found in Fanconi's anaemia and
Bloom's Syndrome).
The immune deficiency is variable, with the commonest being an IgA deficiency.
Thrombocytopenia suggests Wiskott-Aldrich Syndrome, particularly in a boy with immune deficiency and eczema.

Correct Answer. c

Copyright 2014 Delhi Academy of Medical Sciences, All Rights Reserved. 83/85
(197). An immunologic abnormality usually associated with the acquired immunodeficiency syndrome is

a. Decreased levels of IgG

b. Decreased levels of IgA

c. Decreased number of B lymphocytes

d. Anergy in delayed hypersensitivity tests

Solution. Ans 197: (d) Anergy in delayed hypersensitivity tests


Reference: Read the text below
Sol:
Patients with acquired immunodeficiency syndrome have a profound immunologic deficit similar to that associated with DiGeorge
syndrome.
Lymphopenia is usually present along with skin-test anergy because the immunodeficiency is largely within the T lymphocyte population.
Numbers of B lymphocytes usually are normal.
Levels of IgG and IgA usually are elevated in response to opportunistic infections.

Correct Answer. d

(198). Evidence of infection with hepatitis B virus is seen in association with which of the following disease states?

a. Henochschnlein purpura

b. Mixed cryoglobulinemia

c. Wegeners granulomatosis

d. Sjogrens syndrome

Solution. Ans 198: (b) Mixed cryoglobulinemia


Reference: Read the text below
Sol:
Two thirds of patients with mixed cryoglobulinemia have evidence of infection with hepatitis B virus.
This virus also has been found in the serum of 30 to 40% of patients with polyarteritis nodosa.
Because hepatitis B virus has been found in vascular lesions along with immunoglobulin and complement,vasculitis in these individuals
might be due to immune complexes containing the hepatitis antigen.

Correct Answer. b

(199). In restrictive lung disease:

a. FVC is high

b. FEV is high

c. FEV/FVC is high

d. All of these

Solution. Ans-199: (c) FEV/FVC is high


Ref.: Harrison 17th/1588
Sol :
In restrictive lung disease FEV/FVC is high.
FEV1 and FVC are both reduced in restrictive lung disease but the decrease in FEV1 is in proportion to decrease in FVC ands hence the
FEV1/FVC ratio is normal or increased.

Correct Answer. c

Copyright 2014 Delhi Academy of Medical Sciences, All Rights Reserved. 84/85
(200). Finger clubbing is not a usual feature with :

a. Mesothelioma

b. Bronchiectasis

c. Tropical eosinophilia

d. Fibrosing alveolitis

Solution. Ans-200: (c) Tropical eosinophilia


Ref.: Harrison 17th/231
Sol :

Correct Answer. c

Test Answer
1.(c) 2.(d) 3.(d) 4.(c) 5.(d) 6.(c) 7.(a) 8.(d) 9.(c) 10.(a)

11.(c) 12.(a) 13.(d) 14.(a) 15.(c) 16.(b) 17.(a) 18.(a) 19.(d) 20.(b)

21.(a) 22.(a) 23.(c) 24.(d) 25.(c) 26.(d) 27.(d) 28.(d) 29.(d) 30.(d)

31.(d) 32.(c) 33.(d) 34.(a) 35.(b) 36.(c) 37.(a) 38.(b) 39.(a) 40.(b)

41.(a) 42.(d) 43.(d) 44.(d) 45.(c) 46.(d) 47.(b) 48.(a) 49.(d) 50.(a)

51.(a) 52.(a) 53.(b) 54.(a) 55.(a) 56.(c) 57.(d) 58.(c) 59.(a) 60.(d)

61.(d) 62.(c) 63.(c) 64.(d) 65.(b) 66.(d) 67.(b) 68.(c) 69.(a) 70.(a)

71.(d) 72.(b) 73.(b) 74.(b) 75.(c) 76.(a) 77.(d) 78.(b) 79.(b) 80.(c)

81.(b) 82.(c) 83.(c) 84.(a) 85.(d) 86.(c) 87.(d) 88.(b) 89.(a) 90.(c)

91.(b) 92.(d) 93.(a) 94.(a) 95.(a) 96.(b) 97.(d) 98.(d) 99.(d) 100.(d)

101.(d) 102.(c) 103.(a) 104.(d) 105.(a) 106.(d) 107.(c) 108.(d) 109.(a) 110.(d)

111.(c) 112.(a) 113.(d) 114.(a) 115.(d) 116.(d) 117.(a) 118.(c) 119.(c) 120.(a)

121.(b) 122.(c) 123.(d) 124.(c) 125.(d) 126.(c) 127.(b) 128.(c) 129.(d) 130.(a)

131.(c) 132.(b) 133.(d) 134.(d) 135.(d) 136.(b) 137.(b) 138.(b) 139.(d) 140.(c)

141.(d) 142.(b) 143.(b) 144.(c) 145.(c) 146.(a) 147.(c) 148.(c) 149.(d) 150.(c)

151.(c) 152.(d) 153.(d) 154.(c) 155.(c) 156.(b) 157.(d) 158.(a) 159.(a) 160.(b)

161.(d) 162.(a) 163.(c) 164.(a) 165.(c) 166.(b) 167.(d) 168.(c) 169.(a) 170.(b)

171.(a) 172.(a) 173.(c) 174.(d) 175.(a) 176.(d) 177.(d) 178.(a) 179.(c) 180.(a)

181.(b) 182.(c) 183.(d) 184.(b) 185.(a) 186.(c) 187.(a) 188.(b) 189.(d) 190.(b)

191.(b) 192.(c) 193.(d) 194.(c) 195.(c) 196.(c) 197.(d) 198.(b) 199.(c) 200.(c)

Copyright 2014 Delhi Academy of Medical Sciences, All Rights Reserved. 85/85

Вам также может понравиться